Die Quantenmechanik ist schwer zu verstehen, darin sind sich wohl alle einig. (Naja, fast alle – ich kenne jemanden, der das anders sieht und die QM einfacher als die klassische Physik findet…) Und wenn ich hier im Blog über Quantenmechanik schreibe, dann versuche ich jedesmal wieder, bestimmte Grundlagen zu erklären, jedesmal mit leicht anderen anschaulichen Beispielen. Wenn ich mich an die Zeit zurückerinnere, in der ich noch nicht studiert habe, dann war es damals so, dass ich wieder und wieder populärwissenschaftliche Darstellungen lesen konnte, jede war ein bisschen anders, alle waren irgendwie ähnlich, aber keine gab nir das Gefühl, es wirklich verstanden zu haben.

Ich vermute, dass das daran liegt, dass anschauliche Erklärungen (beispielsweise mit Quantentalern) das Problem, haben, dass man sie zwar nachvollziehen kann, man kann sie aber nur schwer auf andere Situationen übertragen und von ihnen aus weiterdenken. (Es ist ja ein häufiges Phänomen in den Diskussioen hier auf dem Blog, dass KommentatorInnen verwirrt sind, weil sie eine anschauliche Erklärung, die eben nur bedingt allgemeingültig ist, weiterzudenken versuchen und dann auf Widersprüche stoßen.)

Physikbücher über Quantenmechanik sind allerdings auf der anderen Seite voll von Mathematik – und man lernt nicht nur etwas über quantenmechanische Zustände, sondern meist auch noch Dinge wie das Lösen von partiellen Differentialgleichungen mit irgendwelchen Ansatzfunktionen, so dass die Physik oft hinter der Mathematik verschwindet.

Gibt es einen Mittelweg zwischen diesen beiden Extrema – Quantentaler auf der einen Seite, Differentialgleichungen auf der anderen? In diesem Text will ich versuchen, so einen Weg zu finden und den Begriff des quantenmechanischen Zustandes einzuführen, und zwar so, dass die Erklärungen keine Analogien sind. Ich habe keine Ahnung, ob das so klappt – deswegen bitte ich vor allem diejenigen, die keine PhysikerInnen sind, um Anmerkungen oder Kommentare. Wo klemmt’s, wo war es unverständlich, wo fehlt etwas? Wenn’s denn einigermaßen klappt, dann hoffe ich, dass ich in ein oder zwei weiteren Teilen dahin komme, auch solche Dinge wie Quantenverschränkung so zu erklären, dass ihr sie selbst wirklich nachvollziehen (und nicht nur staunend ansehen) könnt.

Quantenmechanische Zustände

Ein Teilchen der klassischen Physik ist immer in einem bestimmten Zustand. Es hat beispielsweise einen bestimmten Ort (das Teilchen ist jetzt gerade “hier”) und zusätzlich auch eine bestimmte Geschwindigkeit (es fliegt nach “dort”).

In der Quantenmechanik ist das anders. Betrachten wir zunächst mal das Teilchen nur in einem einzigen Moment – dann müssen wir uns über seine Geschwindigkeit erst mal keine Gedanken machen, sondern können uns auf seinen Ort konzentrieren. Um die Sache noch übersichtlicher zu halten, erlauben wir unserem Teilchen erst einmal nur zwei Möglichkeiten für den Ort – es kann entweder “hier” sein oder “da”. Es hat sich eingebürgert, Zustände in der Quantenmechanik in spezielle Klammern zu schreiben, damit man gleich weiß, dass das, was man meint, ein Zustand ist. Der Zustand “hier” wird geschrieben als |hier>, der Zustand “da” als |da> (und allgemein der Zustand “blubb” als |blubb> ).

In der klassischen Physik gibt es jetzt (in unserem hypothetischen Szenario) genau diese beiden Möglichkeiten für den Zustand des Teilchens: Entweder ist es “hier” oder es ist “da”, eine andere Möglichkeit gibt es nicht.Wenn wir wollen, können wir das mathematisch so aufschreiben:

|Klassischer Zustand> = 1 |hier> + 0 |da>    oder   0|hier> + 1 | da>

Entweder ist der Zustand |hier>, dann multipliziere ich eine Null an den |da>-Zustand (irgendwas mal Null ist ja immer Null, das gilt auch für Zustände), oder es ist andersherum.

Allgemein kann ich das auch so umschreiben:

|Klassischer Zustand> = a |hier> + b |da>  , mit  entweder a=1,b=0 oder a=0,b=1.

Das ist natürlich ziemlich umständlich und ergibt in der klassischen Physik wenig Sinn.

In der Quantenmechanik ist es anders: Es gibt nicht bloß zwei mögliche Zustände eines Teilchens, sondern unendlich viele – es gibt Mischzustände, bei denen das Teilchen sich so verhält, als wäre es gleichzeitig “hier” und “da”. Die kann ich genauso schreiben

1 / 2 / 3 / 4 / 5 / Auf einer Seite lesen

Kommentare (197)

  1. #1 Aasaas
    5. Januar 2013

    Der Artikel ist toll, soetwas hab ich mir schon immer gewünscht!
    Aber mehr Mitdenk-Aufgaben können nie schaden 🙂

  2. #2 Ludger
    5. Januar 2013

    Danke! Ich schau mal, wie weit ich mitkomme.
    Zum Verständnis:
    1.
    Es hat beispielsweise einen bestimmten Ort (das Teilchen ist jetzt gerade “hier”) und zusätzlich auch eine bestimmte Geschwindigkeit (es fliegt nach “dort”).

    Es geht also nicht um den Impuls also um zwei Maße nämlich Ort und Geschwindigkeit?
    2.
    Die Summe der Quadrate aller [Wahrscheinlichkeits-]Amplituden muss genau eins ergeben, warum, sehen wir noch).

    Warum die Quadrate und nicht direkt die Wahrscheinlichkeitsamplituden? Habe ich das übersehen oder kommt es noch?
    3.
    |Zustand> = ψ(x1) |x1> + ψ(x2) |x2>+ ψ(x3) |x3> + …

    Wie liest man das? Wahrscheinlichkeitsamplitude Psi x1 von x1 plus Wahrscheinlichkeitsamplitude Psi x2 von x2 usw?

  3. #3 Maximilian Bremerich
    5. Januar 2013

    @Ludger

    Ich glaube, es funktioniert folgendermaßen:

    1.) Klassisch berechnet sich der Impuls eines Teilchens als Produkt aus Masse und Geschwindigkeit (und semiklassisch als Produkt aus der reduzierten Planckkonstante und der Wellenzahl), das heißt für die Kenntnis des Impulses ist die Kenntnis der Geschwindigkeit erforderlich,

    2.) Weil sich die Aufenthaltswahrscheinlichkeit eines Teilches erst aus dem Quadrat (oder genauer: dem Betragsquadrat) der Amplitude der Wellenfunktion ergibt. Die Amplitude selbst ist nicht die Wahrscheinlichkeit.

    3.) Ich würde es lesen als: Bra Zustand Ket gleich Psi von x1 Bra x1 Ket plus Psi von x2 Bra x2 Ket und so weiter.

  4. #4 MartinB
    5. Januar 2013

    @Aasaas
    Danke, dann werde ich mir im nächsten Teil Mühe geben, noch mehr Mit-Denk-Sachen einzubauen. (Wird auch immer leichter, je mehr sachen schon erklärt sind.)

    @Ludger
    1. Ja. im Moment kümmern wir uns nur um Orte und verschiedene Möglichkeiten dafür.

    2. Wenn du mit warum meinst, warum die Natur gerade so funktioniert und nicht so, dass die Amplituden selbst schon die Wahrscheinlichkeit sind – das weiß niemand.
    Mathematisch würde es deswegen nicht sinnvoll funktionieren, weil Wahrscheinlichkeiten immer positiv sein müssen, die Amplituden aber nicht.
    Das genauer aufzudröseln ist aber etwas für einen späteren Teil (wenn es einem geben sollte). Die Regel als solche muss man einfach akzeptieren. Durch das Quadrieren entstehen erst eine Vielzahl der Quanteneffekte, die wir beobachten, insbesondere ist es für Interferenzphänomene wie den Doppelspalt verantwortlich.

    3. Da das eine selten verwendete Schreibweise ist, gibt es dafür in dieser Form keine wohldefinierte Lesart – ich würde vermutlich sagen:
    Amplitude von x1 mal Zustand x1 plus …
    aber deine Variante ist meiner Ansicht nach auch o.k.

    @Maximilian
    Nein, ein bra ist hier nirgends, das sind alles kets. (Bras sind ja Klammern nach links und leben im dualen Raum). (@alle anderen: Das dürft ihr als technobabble verbuchen…)

  5. #5 Made
    5. Januar 2013

    Wenn ich mit einer Wahrscheinlichkeit von 10% drei richtige im Lotto habe und mit einer Wahrscheinlichkeit von 1% 4 richtige (die Zahhlen stimmen natürlich nicht), dann ist die Wahrscheinlichkeit für 3 oder 4 richtige eben 11%.

    Stimmt das?

    Für 4 richtige sind doch 3 richtige schon mal die Voraussetzung (4 richtige sind sogar eine echte Teilmenge von 3 richtige)? Die Wahrscheinlichkeiten kann so doch nicht einfach addieren. Wenn ich nur mit 10% überhaupt 3 richtige bekommen, kann ich doch nicht mit 11% 3 richtige oder sogar 4 richtige bekommen?

  6. #6 Jakob B.
    5. Januar 2013

    Hallo Martin,
    ich habe schon einiges populärwissenschaftliches über Quantenmechanik gelesen (aber eher von der “Quantentaler”-Art). Ich empfinde die QM als so wenig anschaulich und intuitiv, dass ich schwierigkeiten habe bestehendes Wissen abzurufen und auf konkrete Aufgabenstellungen anzuwenden. Wenn mir also ein versuchsaufbau präsentiert wird den ich noch nicht gesehen habe kann ich nicht entfernt vorhersagen was wohl passieren wird (bei der klassischen Mechanik gelingt mir dies bis zu einem gewissen Punkt). Die Probleme, die die bedingte Allgemeingültigkeit anschaulicher Erklärungen mit sich bringen, kann ich also bestätigen.
    An dieser Stelle hat mir der Artikel sehr weitergeholfen. Grundsätzlich war mir das Meiste schon vorher klar, es wurde aber so präsentiert und auf den Punkt gebracht, dass aus unzusammenhängenden Informationen ein gewisses Verständnis wachsen kann.

    An dem Punkt 2 von Ludger bin ich auch hängen geblieben. Innerhalb des Artikels wird immer nur das Quadrat der Wellenfunktion verwendet. Es erscheint ohne weitere Informationen oder Anwendungen (d.h. Formeln in denen die (evtl. negative) Wurzel der Wahrscheinlichkeit verwendet wird) nicht sinnvoll. Ich freue mich auf hoffentlich folgende Artikel, die das klären.

    Wenn ich mich hier schonmal zu Wort melde, ergreife ich gleich die Gelegenheit Dich mal gründlich zu loben. Durch Deinen Blog habe ich nicht nur sehr viel über Physik und Wissenschaft allgemein gelernt, vorallem hast Du mein interesse für Philosophie geweckt. “Zen und die Kunst ein Motorrad zu warten” habe ich nach deiner Empfehlung gelesen und verschlungen und derzeit lese ich Kant (bin
    mir noch nicht ganz sicher ob ich mich da nicht überforder), als nächstes ist Wittgenstein dran. Vielen Dank für die Inspiration, weiter so!

  7. #7 Jakob B.
    5. Januar 2013

    @Made:
    es sind 10% Wahrscheinlichkeit für “genau 3 richtige” gemeint und nicht für “mindestens 3 richtige”.

  8. #8 Made
    6. Januar 2013

    @Jakob B.
    Ja, so macht das Sinn. Hab ich mir nach abschicken meines Kommentars dann auch gedacht.

  9. #9 Huebi
    6. Januar 2013

    Richard Feynman:I think I can safely say nobody understands Quantummechanics“ – The Character of Physical Law. MIT Press, 1967

    🙂

  10. #10 Daniel K
    6. Januar 2013

    Der Artikel ist richtig gut gelungen, vorallem erfährt man etwas mehr von der Mathematik dahinter!

    Es wäre jedoch nett wenn du in Zukunft öfters LaTex benutzen würdest, auch bei sowas:
    |QM-Zustand> = a |hier> + b |da> + c | dort>,
    Formeln lesen sich so mMn viel besser als wenn sie einfach in den Text eingebettet sind.

  11. #11 Rumo
    6. Januar 2013

    Sehr schöner Artikel, die – zumindest für mich – den perfekten Mittelweg zwischen Anschaulichkeit und Formeln findet. Freue mich schon sehr auf eine Fortsetzung 🙂

    Zu deinen Fragen: Ein paar weitere Mitdenkaufgaben wäre auf jeden Fall noch eine Verbesserung. An solchen kann man mMn immer am besten erkennen, ob man den Sachverhalt schon verstanden hat oder nicht.
    Zu der Mitdenkaufgabe in diesem Artikel hätte ich auch gleich nochmal ne Frage: Ist es Absicht, dass bei d) viermal der gleiche Zustand als Summand erscheint? (falls ja, wäre es evtl. hilfreich darauf in der Antwort noch kurz einzugehen 🙂

  12. #12 MartinB
    6. Januar 2013

    @JakobB
    Das mit dem Quadrat der Amplituden wird vermutlich im zweiten Teil klarer – wie ich den aufziehen muss, ist mir heute nacht eingefallen (dafür habe ich leider etwas Schlaf eingebüßt, aber so ist das, wenn die Gedanken sich festbeißen).

    Kant lesen ist schon ziemlich heftig, weil die Sprache wirklich böse ist. Viel Glück dabei.

    @Huebi
    Das bezieht sich auf die Interpretation der QM – nicht auf den physikalischen Apparat, den versteht man sehr gut (auch und gerade Feynman tat das, siehe seine Lectures).

    @DanielK
    LaTeX hat den Nachteil, dass hier das WordPress-Latex-Plugin nicht geht, d.h. ich muss alle latex-Formeln mühsam über codecogs einbetten, deswegen spare ich mir die Mühe dort, wo es zur Not auch so geht.

    @Rumo
    Äh, nein, das war eigentlich keine Absicht, es sollten die Zustände 1-4 sein. So wie es da steht, ist es aber auch o.k. – ich darf auch zweimal denselben Zustand addieren (nur nicht multiplizieren).

  13. #13 MartinB
    6. Januar 2013

    @Rumo
    So, ich hab’s korrigiert, danke.

  14. #14 Bloody Mary
    6. Januar 2013

    Das von Dir geschilderte frustrierende Gefühl, auch nach umfassender Lektüre nur ein vages Verständnis für einen fachfremden Gegenstand entwickeln zu können, kenne ich leider gut. Aber weil man nie aufgeben darf, lese ich auf den scienceblogs mit.

    Dein didaktisches Können finde ich herausragend, bisher hat es noch niemand geschafft, mich ohne meine Gegenwehr an das heran zu führen, was ich als den mathematischen Abgrund empfinde, und mich sogar zu veranlassen, ihn auf dem Seil, das Du für uns gespannt hast, überqueren zu wollen.

    Andere haben es schon vor mir geäußert, aber es drängt mich, es zu wiederholen: dass Du Übungsaufgaben zur eigenständigen Verständniskontrolle anbietest, ist Klasse.

    Ich freu mich schon auf die Fortsetzung (oder, falls es keine geben sollte, auf Deinen nächsten Beitrag).

  15. #15 rolak
    6. Januar 2013

    ohne meine Gegenwehr

    hübsch 🙂

    btw: Ist das Kater Murr?

  16. #16 segeln141
    6. Januar 2013

    deswegen bitte ich vor allem diejenigen, die keine PhysikerInnen sind, um Anmerkungen oder Kommentare. Wo klemmt’s, wo war es unverständlich

    Für mich immer noch zuviel Mathematikformeln.

    [quote]Ein Teilchen der klassischen Physik ist immer in einem bestimmten Zustand. Es hat beispielsweise einen bestimmten Ort (das Teilchen ist jetzt gerade “hier”) und zusätzlich auch eine bestimmte Geschwindigkeit (es fliegt nach “dort”).

    In der Quantenmechanik ist das anders. Betrachten wir zunächst mal das Teilchen nur in einem einzigen Moment – dann müssen wir uns über seine Geschwindigkeit erst mal keine Gedanken machen, sondern können uns auf seinen Ort konzentrieren. Um die Sache noch übersichtlicher zu halten, erlauben wir unserem Teilchen erst einmal nur zwei Möglichkeiten für den Ort – es kann entweder “hier” sein oder “da”. Es hat sich eingebürgert, Zustände in der Quantenmechanik in spezielle Klammern zu schreiben, damit man gleich weiß, dass das, was man meint, ein Zustand ist. Der Zustand “hier” wird geschrieben als |hier>, der Zustand “da” als |da> (und allgemein der Zustand “blubb” als |blubb> [/quote]

    [quote]In der klassischen Physik gibt es jetzt (in unserem hypothetischen Szenario) genau diese beiden Möglichkeiten für den Zustand des Teilchens: Entweder ist es “hier” oder es ist “da”, eine andere Möglichkeit gibt es nicht.Wenn wir wollen, können wir das mathematisch so aufschreiben:[/quote]
    bis zum Satz
    [quote]Wenn wir wollen, können wir das mathematisch so aufschreiben[quote] fand ich das gut,bzw.habe gehofft es geht weiter mit weniger Mathe und mehr erklärenden Sätzen.

    Lieber MartinB: wenn das nicht geht,ist dies nur meinem beschränkten Verständnis zuzuschreiben.Oder können Physiker nur mit Formeln arbeiten,wenn sie allgemeinverständlich etwas beschreiben wollen?.
    Allgemeinverständlich ist das entscheidenede Wort,dass Physik nur mit mathematischen Gleichungen/Formeln arbeiten muss,ist mir schon klar.
    Zur Zeit lese ich das Buch über Paul Dirac: “The strangest Man”,das mir FF empfohlen hatte.Super gut,Der narrative Stil läßt die Persönlichkeit Dirac`s sehr plastisch werden.
    Er hat ja ein binary vocabulary (ja/nein).
    Warum gibt es hier keine Kommentarvorschau,damit man Fehler besser korrigieren kann?

  17. #17 segeln141
    6. Januar 2013

    Addendum :ohne Kommentarvorschau konnte ich nicht erkennen,dass Zitate nicht mit [quote] [/quote] als Zitate erscheinen,sondern nur mit

  18. #18 segeln141
    6. Januar 2013

    zitat

  19. #19 MartinB
    6. Januar 2013

    @segeln
    Das mit den Formeln ist immer eine Gratwanderung. Man kann diese Dinge auch versuchen, komplett formelfrei aufzuschreiben – das problem ist nur, dass es dann sehr unüberischtlich wird, wie du z.B. an dem Beispiel mit der Formel siehst, nach deren Lesart Ludger gefragt hat.
    Eine Formel wie die unter den Beispielaufgaben bei d) wäre in Worten zwar auszudrücken, aber sehr unübersichtlich.
    Die Formeln hier sind aber nur eine Art “Sprache” – ich verlange ja nicht von jedem, der hier mitliest, dass er oder sie irgendwas nachrechnet. Es ist ungefähr so als würde ich dir etwas in der Biologie erklären – manchmal muss man Begriffe wie “Postzygapophyse” einfach lernen, weil es sonst einfach zu lange dauert immer wieder “Fortsatz hinten am Wirbel….” zu sagen. Genau so kannst du hier die Formeln lesen:
    “Der Zustand ist eine Überlagerung aus den Zuständen hier und da…”

    Es ist eine Frage der Effizienz, und wenn man QM wirklich verstehen will, dann ist es praktisch, sich auf die Formelschreibweise einzulassen, einfach weil sie so kurz ist. (Andere Analogie: Klar könnte ich beim Schachspiel auch alle Züge beschreiben: Der Springer, der rechts am Königsflügel entwickelt ist, geht auf das Feld in der Mittellinie im Schwarzen Bereich – aber Sf3-e5 geht einfach schneller.)

    Lass dich von den Formeln also nicht abschrecken.

  20. #20 Igambris
    6. Januar 2013

    Sehr guter Versuch!
    Für mich liest es sich flüssig bis zu dieser Stelle
    im Abschnitt Wellenfunktion, wo ich beim schnellen Lesen
    ins Stolpern geraten bin:
    “In ψ(x) steckt jetzt also die Information darüber,…”
    Ich bin am überlegen, ob diese Fassung besser wäre:
    ==========================================
    Der Gesamtzustand des Teilchens ist dann also:
    |Gesamtzustand> = ψ(x1) |x1> + ψ(x2) |x2>+ ψ(x3) |x3> + …
    und zwar summiert über alle möglichen Werte von x.
    (Warnschild) … (Warnschild Ende)
    “|Gesamtzustand>” wird Wellenfunktion genannt
    Anstelle von |Gesamtzustand> schreibt man |ψ(x)> oder auch ψ(x).
    ψ(x) ist einfach eine andere Schreibweise für den Gesamtzustand.
    Diese Schreibweise ist sinnvoll, weil die Wellenfunktion ψ(x)
    einen Wert an jedem Punkt des Raumes hat, und man
    sich das Hinschreiben der Summe der Ortszustände auch sparen kann.
    In ψ(x) steckt jetzt also die Information darüber,
    mit welcher Wahrscheinlichkeit das Teilchen wo ist
    – die Wahrscheinlichkeit für den Ort x ist gegeben durch ψ(x)²….
    =============================================
    Ich habe diesen Artikel übrigens auf meiner Seite
    http://www.josef-graef.de/zukunft/qputer.html verlinkt.
    Spätestens seit dem Erscheinen dieses didaktischen Glanzstücks:
    https://scienceblogs.de/hier-wohnen-drachen/2012/07/14/kann-man-das-higgsteilchen-verstehen/
    bin ich ein begeisterter Leser von “Hier wohen Drachen”.

  21. #21 MartinB
    6. Januar 2013

    @lgambris
    Das ist in meinen Augen eben nichtganz korrekt – der Zustand wird nicht Wellenfunktion genannt. Die Wellenfunktion enthält die Amplitude für jeden Ort, ist aber selbst eigentlich kein Zustand. Man kann sie eindeutig einem Zustand zuordnen. Ich fand es früher jedenfalls immer verwirrend, dass manchmal so getan wird, als sei die Wellenfunktion der Zustand und manchmal wird sie als Amplitude an einem Ort behandelt – dieser Verwirrung wollte ich vorbeugen.

  22. #22 Harg
    Meissen
    6. Januar 2013

    Toller Anfang, für mich genau die richtige Balance zwischen einfach verständlich und auf die physikalisch/mathematischen Grundlagen eingehend.

    Mein persönliches Verständnis-Problem bei der Quanten-Mechanik ist, dass mir immer folgende (falsche) Trivial-Erklärung im Kopf rumspukt: die Wellenfunktion beschreibt keine Ueberlagerung des Teilchen-Zustandes, sondern gibt einfach die Wahrscheinlichkeit für einen Zustand an (z.B. den Ort, wo sich das Teilchen tatsächlich aufhält). Also das Teilchen ist an einem bestimmten Ort, den wir aber bis zur Messung nicht genau kennen, wir können nur eine Wahrscheinlichkeits-Aussage machen. Die Messung wäre dann kein Zusammenbruch der Wellenfunktion, sondern lediglich die Konkretisierung, wo in diesem Falle das Teilchen tatsächlich gesteckt hat. Und wenn man dann eben z.B. 1000 Messungen gemacht hat, bekommt man eine Verteilung der gemessenen Orte, wie sie die Wellenfunktion angibt. Mit dieser Trivial-Deutung hätten sich die meisten Probleme und Rätsel der Quanten-Mechanik erledigt – nur leider: die Erklärung ist falsch, das ist mir durchaus bewusst. Ich habe bisher aber leider in keinem (populärwissenschaftlichen) Aufsatz eine Widerlegung für diese triviale Deutung gefunden. So hoffe ich für mich persönlich, dass durch die Artikelreihe dieser Spuk endlich aus meinem Kopf kommt und ich zu einer besseren Einsicht in die Natur der Quanten-Mechanik komme.
    Jedenfalls bin ich schon sehr auf die nächsten Folgen gespannt und werde sie sorgfältig durcharbeiten.

  23. #23 MartinB
    6. Januar 2013

    @Harg
    Die Widerlegung der Trivial-Deutung steckt im EPR-Paradoxon bzw. in der Bellschen Ungleichung. Beide beruhen auf der Quantenverschränkung – da der Artikel hier ja anscheinend ganz gut ankommt, wird es noch weitere teile geben, vermutlich brauche ich noch zwei, bis ich bei der Verschränkung ankomme.
    Es gab dazu auch mal eine Serie auf dem alten Blog artefakten:
    https://scienceblogs.de/arte-fakten/2010/06/18/die-eprgeschichte-teil-1/
    Auch wenn ich mit Jörg Friedrich meist nicht einer meinung bin (siehe auch die Diskussionen), die Serie war in weiten Teilen ganz gut.

  24. #24 Igambris
    6. Januar 2013

    @MartinB
    Ja, stimmt. Da fällt mir noch ne Fassung ein:
    =================================
    In ψ(x) steckt jetzt also die Information darüber,
    mit welcher Wahrscheinlichkeit das Teilchen wo ist
    – die Wahrscheinlichkeit für den Ort x ist gegeben durch ψ(x)².
    ψ(x) wird Wellenfunktion genannt.
    Weil die Wellenfunktion einen Wert an jedem Punkt des Raumes hat,
    kann man sich das Hinschreiben der Ortszustände |xi> auch sparen
    und direkt sagen, dass z.B. ψ(x1) den Zustand |x1> kennzeichnet.
    Manchmal wird die Schreibweise auch noch weiter
    verzerrt und man schreibt |ψ(x)>

  25. #25 MartinB
    6. Januar 2013

    @lgambris
    Ehrlich gesagt sehe ich jetzt keinen großen Unterschied mehr zwischen unseren Fassungen.

  26. #26 Igambris
    6. Januar 2013

    @MartinB
    Ich habe nur laut gedacht, wie ich den einzigen Stolperstein des ganzen Artikels, über den ich bei der Schnell-Lektüre gestrauchelt bin, wegräumen würde. Vielleicht bin ich ja einzige, der an dieser Stelle stockte, dann ist ja alles gut.

  27. #27 Bloody Mary
    6. Januar 2013

    @rolak
    Erst mal danke, dass Du mir deutlich weniger schwierige Fragen als MartinB stellst:

    „ist das Kater Murr?“

    Nein, das bin doch ich, Cat-Sin, beim Schreiben 🙂
    Und jetzt die seriöse Antwort:
    auf der Postkarte hinten drauf steht „The Animal Kingdom, AK. 170 „Thinking Of You“, Reproduced from an illustration dated 1906 (Private Collection).“. Das Motiv gefiel mir wegen der Schreibfeder.

  28. #28 Harg
    6. Januar 2013

    @MartinB
    Danke für die Antwort: Die Stichworte EPR-Paradoxon und Bell-Ungleichung führten wirklich genau ins Zentrum meines Unverständnisses / meiner Fragen. Gleich bei Wiki nachgeschlagen und sehr überrascht zu lesen, dass da solch fundamentale Fragen wie die Lokalität und Realität des physikalischen Weltbildes eine Rolle spielen – und dass sogar ein Genie wie Einstein sich die letzten 20 Jahre seines Lebens damit herumschlug. Naja, die Details sind mir allerdings nicht ganz klar geworden, um so “heisser” bin ich nun auf die Fortsetzung Deiner Einführung in die QM.

  29. #29 Ludger
    6. Januar 2013

    segeln141
    6. Januar 2013
    Addendum 😮 hne Kommentarvorschau konnte ich nicht erkennen,dass Zitate nicht mit [quote] [/quote] als Zitate erscheinen,sondern nur mit

    Nimm statt der eckigen Klammern “[ “bzw. “]” die Größer- bzw Kleinerzeichen “” und statt “quote” das Wort “blockquote”, dann gehts.

  30. #30 MartinB
    6. Januar 2013

    @segeln
    Ich habe nochmal nachgedacht und würde gern folgendes fragen, damit ich das problem didaktisch besser in den Griff bekomme:
    Warum genau ist es für dich schwieriger, die Formel
    a|hier> + b |da>
    zu verstehen als die Aussage
    “Die Wahrscheinlichkeitsamplitude hat einen Wert für “hier”, einen anderen für “da”)?
    Kannst du das an irgendwas festmachen? Ist das einfach ein genereller Mathematik-Horror, ist es die implizite Interpretation der Formelzeichen (z.B., dass das a zu “hier” gehört, weil es als Faktor steht und ja Punktrechnung vor Strichrechnung geht) oder ist es irgendwas anderes? (Oderkannst du es nicht an etwas festmachen?) Würde mir echt weiterhelfen, sowohl hier beim Schreiben als auch sonst, wenn ich Vorlesungen halte, wenn ich genauer verstehen würdfe, was das Problem ist.

    Was die Kommentarvorschau betrifft: Das neue WordPress-System wird vermutlich von einem Praktikanten betreut, der sich eigentlich bei National Geographics als Zoowärter beworben hatte und noch nie einen rechner gesehen hat, deswegen klappt hier alles nicht so wie es soll.

  31. #31 Bloody Mary
    6. Januar 2013

    @MartinB
    Ich war zwar gar nicht gefragt, aber bei mir ist es der generelle Mathematik-Horror. Sobald Formeln ins Spiel kommen, stürzt automatisch mein CPU ab.Ich kann nicht mal mehr hinschauen oder -hören, die Augen werden einfach zugekniffen und wenn ich sie wieder öffne, soll das Ding bitte, bitte weg sein! Dank Deiner sonderpädagogischen Begabung konnte dieses Reiz-Reaktionsschema diesmal außer Kraft gesetzt werden.

    Übrigens fand ich die Aggressivität Deines Zoowärter-Scherzes sehr gewinnend, Du kannst wirklich temperamentvoll werden, wenn es angemessen ist, genau wie noch jemand hier, den ich sehr mag.

  32. #32 roel
    *****
    6. Januar 2013

    @MartinB Ich kann mich eigentlich nur immer wiederholen: Super Beitrag!

    Ein kleines Problem habe ich mit dieser Formulierung: “…und zusätzlich auch eine bestimmte Geschwindigkeit (es fliegt nach “dort”).” Dort ist die Richtung und nicht die Geschwindigkeit.

  33. #33 MartinB
    6. Januar 2013

    @Bloody Mary
    Danke für den Hinweis mit dem Mathe-Horror. Schön, wenn er prinzipiell überwindbar ist.
    Was den Zoowärter-Spruch angeht: “Tact is for people not witty enough for sarcasm…”

    @roel
    Naja, die geschwindigkeit steckt im “fliegt”.

  34. #34 rolak
    6. Januar 2013

    zumal die Richtung ein nicht unwesentlicher Teil einer Geschwindigkeit ist, so ein Vektor ohne Richtung ist ja nur wieder ein Punkt.

  35. #35 Kallewirsch
    6. Januar 2013

    Also ich finde die Mathe hier nicht so schlimm. BIs jetzt ist das für mich noch nachvollziehbar (mathematisch).

    Dinge, bei denen ich regelmässig verständnismässig aussteige

    Das mit dem Quadrat der Amplituden wird vermutlich im zweiten Teil klarer

    Ja, diese Quadrate haben mich schon immer etwas verwirrt. Ich hatte immer das Gefühl, die werden irgendwie aus der Luft herbeigezaubert. Für mich hoffe ich mal, das das noch klarer wird.

    Das andere, was mich immer extrem verwirrt:
    Woher wissen wir eigentlich, dass vor der Messung alle Zustände gleichberechtigt vorliegen und sich das System erst dann stabilisiert, wenn gemessen wird?
    Damit hab ich Schwierigkeiten. Wie kann ich eine Aussage über das Verhalten VOR einer Messung machen, wenn ich durch die Messung das System beeinflusse. Tue ich das überhaupt? Kann es nicht sein, dass die verschränkten Teilchen nicht grundsätzlich in einem der beiden Zustände entstehen (mit einer gewissen Wahrscheinlichkeit) und ich dann eigentlich nur das Messe, was seit Entstehung der Teilchen eh schon feststand.
    (Ja ich weiß, du wirst jetzt sagen: genau das ist nicht so, Das ist ja das Geheimnisvolle an der QM). Aber ich hab da so meine Schwierigkeiten. Wenn du also so lieb wärst, in den Folgeartikeln auch ein wenig praktisch zu begründen, warum die Dinge so sind (mit welchem Experiment kann man das feststellen), dann wäre mir zumindest ein ganzes Stück weiter geholfen. Danke gleich vorab.

  36. #36 roel
    *****
    6. Januar 2013

    @MartinB und @rolak ja, alles klar.

    Und jetzt zu den Fragen.
    “Hat die Erklärung was genützt?” Ja klar, du erklärst das mit einer Leichtigkeit, da kann ich nur staunen.
    “Wisst Ihr mehr als vorher?” Genau aus diesem Grund lese ich hier diese Beiträge, um einen Einblick in die Physik zu erhalten.
    “Bräuchtet ihr noch mehr Beispiele” Beispiele sind immer willkommen.
    “oder kleine Mitdenk-Aufgaben?” Das ist eine super Idee und ich denke da stimmen fast alle überein, das kannst du sehr gerne beibehalten!

  37. #37 MartinB
    7. Januar 2013

    @Kallewirsch
    Ja, das zu erklären war ursprünglich mein Ziel, als ich anfing zu schreiben (und dann fing ich mit Grundlagen an und merkte, dass es doch länger dauert, dass wirklich zum Mitdenken zu erklären).
    Ein Fall, an dem man sehen kann, dass der Zustand eines Quantensystems nicht schon festliegt, ist der Doppelspalt, ein anderes Beispiel habe ich mal ausführlich hier diskutiert:
    https://scienceblogs.de/hier-wohnen-drachen/2011/05/29/quantenmechanik-nichtlokalitat-und-unscharfe/
    Da sieht man auch, dass der Zustand eben nicht schon festliegen kann.

  38. #38 MartinB
    7. Januar 2013

    Nachtrag: Ich sehe gerade, dass der Artikel dank unseres Server-Umzugs fast unlesbar gewirden ist, weil die ganzen Pfeile alle durch Fragezeichen ersetzt wurden. Gut gemacht, NatGeo.

  39. #39 schlappohr
    7. Januar 2013

    Ausgezeichnet. Ich habe genau das Problem, das Du beschrieben hast: Viele populärwissenschaftliche Bücher gelesen, auch sehr gute (Green, Hawking, Randall), aber es bleibt immer etwas Ratlosigkeit, weil man ohne Mathematik nicht weit kommt. Ausnahme ist die “skurrile Quantenwelt” von Silvia Arroyo Camejo, die den gleichen Spagat versucht wie Du, was ihr auch sehr gut gelungen ist.
    Ich bin gespannt auf die nächsten Teile.

  40. #40 Ulf Lorenz
    7. Januar 2013

    @Igambris:

    Ein kleines Detail noch: Man schreibt ueblicherweise in Kurzform |psi> (ohne das “(x)”) fuer den Gesamtzustand. Denn |psi> ist ein Zustand, waehrend psi(x) eine Funktion der Koordinate “x” ist; man kann auch andere Koordinaten y waehlen (z.B. Winkel statt kartesischer Koordinaten bei mehreren Dimensionen); dann benoetigt man eine andere Funktion psi(y) der Koordinaten, meint aber eigentlich denselben Zustand |psi>.

  41. #41 segeln141
    7. Januar 2013

    @Ludger
    6. Januar 2013
    Danke für den Hinweis,dies ist das altbekannte Blockquote.
    ich hatte es mal l anders versucht,da ich zur Zeit
    auf einem Portal unterwegs bin,wo das so gemacht wird.
    Die fehlende Kommentarschau ist allerdings etwas ärgerlich

  42. #42 segeln141
    7. Januar 2013

    @MartinB
    6. Januar 2013
    Danke,dass Du Dir nochmal Gedanken gemacht hast.Ich habe es gerade gelesen und möchte sofort kurz antworten.Wenn ich mir das nochmal genauer angeschaut habe werde ich Dir über meine Verständnisprobleme berichten.
    Ansonsten:ja,ja der Zoowärter

  43. #43 MartinB
    7. Januar 2013

    @Ulf
    Danke für den Hinweis, ich hab die Variante noch nachgetragen.

  44. #44 segeln141
    7. Januar 2013

    @MartinB

    kleiner Zwischenbericht,da mir auch der Artikel über die Quantentaler sehr gur gefallen hat.Sicher wirst Du sagen,der war ja total einfach erklärt für entsprechend “ungeeignete Personen”.
    Also für mich war er sehr schön und locker/lustig geschrieben.
    Beim Artikel hier bin ich bis zur ersten Aufgabe ganz gut mitgekommen.Mal sehen ,wie es weiter geht.
    Es war so,dass ich bei Mathe-Formeln beim ersten durchlesen vor 2 Tagen einfach etwas abgeschaltet habe,wobei ich auf dem Gymnasium durchaus gut war in Mathe,aber das ist schon 40 Jahre her.
    Deine mir gegenüber aufmunternde Art und insbesondere das erneute Nachdenken fand ich sehr,sehr gut und “inspiring”
    Ich habe halt den Satz

    deswegen bitte ich vor allem diejenigen, die keine PhysikerInnen sind, um Anmerkungen oder Kommentare. Wo klemmt’s, wo war es unverständlich,

    ernst genommen,da ich keine Scheu habe ,auch mal mein Unwissen erkennen zu geben.

  45. #45 segeln141
    7. Januar 2013

    Aus dem Quantentalerartikel: .

    Fazit: Beeinflusst jede Messung das Ergebnis? Nein. Messungen können den Zustand eines Quantensystems beeinflussen – aber sie tun das nicht immer, sondern abhängig davon, in welchem Zustand das System ist und was ich messe.

    und hier:

    Führt man eine Messung an einem Quantenobjekts durch, dann beeinflusst diese Messung den Zustand so, dass das Objekt hinterher in dem gemessenen Zustand ist. Klingt kompliziert?

    ja,klingt kompliziert,wenn man die Aussage im Quantentalerartikel sich vor Augen hält.
    MarinB was ist die Lösung?

  46. #46 Kallewirsch
    7. Januar 2013

    @segeln141

    und hier:

    Führt man eine Messung an einem Quantenobjekts durch, dann beeinflusst diese Messung den Zustand so, dass das Objekt hinterher in dem gemessenen Zustand ist. Klingt kompliziert?

    ja,klingt kompliziert,wenn man die Aussage im Quantentalerartikel sich vor Augen hält.
    MarinB was ist die Lösung?

    Das find ich relativ leicht einzusehen (nachdem ich den von Martin freundlicherweise für mich verlinkten Artikel gelesen habe). Erzeuge ich senkrecht polarisierte Photonen und schicke die auf ein Polfilter welches 45% gedreht ist, dann kommt das Photon nur mit einer Wahrscheinlichkeit von 50% durch. Aber von denen die durchkommen, weiß ich, wie sie polarisiert sind.

    @Martin
    Könntest du einen Artikel zu dem hier schreiben:
    https://www.mpg.de/6769805/negative_absolute_temperatur

    Irgendwie kommt mir das jetzt wie ein mathematischer Taschenspielertrick vor. (Aber ich hab auch den Zugang aus der klassische3n Physik). Ich seh nicht ein, warum man da von einer negativen Temperatur sprechen kann. Der Taschenspielertrick besteht für mich darin, dass man aus der “Temperatur” die Bolzman-Verteilung herleitet, die eine bestimmte Kurve zeigt. Dreht man jetzt mit Gewalt diese Kurve um, dann kann man doch nicht den umgekehrten Weg gehen und daraus der Temperatur das Attribut negativ herleiten. Ich hab mir eine Analogie einfallen lassen: In Österreich wohnen in den Alpentälern Menschen. Man kann (Achtung: Annahme) sicherlich eine Formel herleiten, die angibt in welcher Höhe über dem Talboden wieviele Menschen leben. Das ergibt eine Formel die diesen Zusammenhang beschreibt. Verfrachte ich jetzt aber alle Bewohner mit Autobussen auf die Berggipfel, dann ändert das aber nichts an der Bevölkerung an sich. Sie ist jetzt nur anders verteilt. Und nur, weil ich für die Hinrechnung eine Formel habe, heißt das noch lange nicht, dass ich die auch für die Rückrechnung nehmen kann. Lasse ich den Dingen ihren natürlichen Lauf (die Leute leben, wo sie wollen), dann mag die Rückrechnung durchaus sinnvoll sein, sobald ich da aber gewaltsam eingreife, stimmt das doch hinten und vorne nicht mehr, wenn ich einfach mein zuvor gewonnene Formel einfach in die Rückrichtung umforme.

  47. #47 Ludger
    7. Januar 2013

    Ich habs mir immers so vorgestellt:
    Wenn die Polizei Ort und Geschwindigkeit meines PKW messen will, nimmt sie langwelliges “Licht”, nämlich Radiowellen (RADAR). Mein Auto ist im Vergleich zu den Radar-Photonen so schwer, dass die Messung an meinem Auto nichts verändert (nur an meinem Konto – da gibt es eine sehr böse Landrätin in der Nähe von Kassel 🙁 ). Will man Messungen an Quantenobjekten machen, braucht man kurzwelliges Licht, weil Quantenobjekte sehr (!) klein und leicht sind. Die Lichtquanten von kurzwelligem Licht sind aber energiereicher als die von langwelligem Licht, nämlich proportional zur Lichtfrequenz ( https://de.wikipedia.org/wiki/Wirkungsquantum ). Je kleiner die zu messenden Objekte sind um so energiereicher weil kurzwelliger muss das zur Messung eingesetzte Licht sein. Irgendwann kommt bei kleiner werdenden Quantenobjekten die Situation, dass die zur Messung eingesetzten Lichtquanten das Messobjekt bei der Messung “stoßen” und dabei dessen Geschwindigkeit oder Ort verändern.
    Wahrscheinlich habe ich gerade nur eine klassich-physikalische Verständniskrücke formuliert. Mal sehen, wie sie mit der in Martins Blog beschriebenen Quantenrealität übereinstimmt.

  48. #48 segeln141
    7. Januar 2013

    @MartinB

    braucht man mehrere Teilchen, deren Zustände miteinander gekoppelt werden – das ist die berühmte “Verschränkung”,

    braucht man “mehrere” Teilchen oder genügt bereits ein Teilchenpaar?
    Ist diese “spuckhafte” Fernwirkung schon erklärbar?

  49. #49 segeln141
    7. Januar 2013

    @Kallewirsch
    7. Januar 2013
    ja,in der Tat,wenn ich den Dir von MartinB verlinkten Artikel gelesen habe ist es sogar mir verständlich geworden.

  50. #50 segeln141
    7. Januar 2013

    @Ludger

    Irgendwann kommt bei kleiner werdenden Quantenobjekten die Situation, dass die zur Messung eingesetzten Lichtquanten das Messobjekt bei der Messung “stoßen” und dabei dessen Geschwindigkeit oder Ort verändern

    ja,so kann man sich das vorstellen,wenn MartinB uns nicht eines Besseren belehrt.

  51. #51 Theres
    leicht OT ;)
    7. Januar 2013

    @Kallewirsch
    Nicht mathematisch, aber via Definition der Temperatur … und schau mal hier nach: https://arxiv.org/pdf/1211.0545v1.pdf Vielleicht hilft dir das weiter. Das ist das Paper zur Veröffentlichung über die negativen Temperaturen. Ich verstehe allerdings dein Beispiel nicht, und der mir bekannte Physiker, mit dem ich vorhin noch darüber diskutierte, hat sich gen Heimat getunnelt (mittels U-Bahn), deshalb wage ich es nicht, mich an ner Erklärung zu versuchen.

  52. #52 Igambris
    7. Januar 2013

    @Ulf
    Ich glaube, jetzt weiß ich, warum ich beim Lesen ins Stocken geraten bin:
    Ich hatte wohl den Begriff “Kollaps der Wellenfunktion” im Hinterkopf
    und zwar so ähnlich wie er in der Wikipedia steht:
    https://de.wikipedia.org/wiki/Kollaps_der_Wellenfunktion
    Das suggeriert, den Gesamtzustand als “Wellenfunktion” zu bezeichnen.
    Im obigen Kontext ist die Wellenfunktion das aber gerade nicht,
    sondern die Wellenfunktion ist eher das “Gewicht” ψ(x).
    Der Begriff “Zustandsreduktion” ist wohl besser als “Kollaps der Wellenfunktion”, klingt aber nicht so gut.
    Und ja, Du hast recht, man schreibt |ψ>. Und wenn man vornehm sein wollte, dann würde man auch “die Funktion ψ” sagen anstelle von “die Funktion ψ(x)”.

    Vor 20 Jahren hatte ich mich zuletzt mit diesen Dingen beschäftigt.
    Seit Martins Higgs-Artikel fange ich an, mich wieder dafür zu interessieren.
    Martin gelingt hier in diesem Blog etwas Phänomenales:
    Es macht richtig Spaß, sich mit mit der QM zu beschäftigen!
    Ich glaube, da kommt weder das beste Lehrbuch,
    noch das beste populärwissenschaftliche Buch mit.
    Es geht hier um ganz große Kunst: Didaktik der Physik

  53. #53 segeln141
    7. Januar 2013

    @MartinB

    nachdem Du geschrieben hast,ich soll mich von den Formeln nicht abschrecken lassen,bin ich tollkühn noch einmal den Text durchgegangen und siehe da,sogar ich kann ihm folgen.
    Ich sehe das so: die Grundlagen für das Verständnis derQM sind eben zunächst mathematisch zu beschreiben.Ich hatte wahrscheinlich zu früh gehofft,etwas praktisches “Anschauungsmaterial” zu lesen,wie in dem Quantentalerartikel.
    Ich glaube und hoffe,dass dies noch kommt.
    Demnach:falls meine Gedanken richtig sind,hast Du didaktisch nichts “falsch” gemacht.Vielleicht eine kleine Vorbemerkung,dass zunächst die mathematischen Grundlagen erklärt werden und dann etwas”Praktischeres” folgen wird.
    Sind meine dahingehenden Überlegungen richtig?
    Auf jeden Fall:ein schöner und guter Artikel bislang !

  54. #54 segeln141
    7. Januar 2013

    und man sollte sich wirklich von mathematischen Formeln nicht prima vista abschrecken lassen

  55. #55 MartinB
    8. Januar 2013

    @segeln
    Nochmal zum scheinbaren Widerspruch mit dem Quantentaler:
    Wenn ich eine bestimmte Messgröße messe, dann sit das Teilchen hinterher in diesem Zustand. Meist führt das zu einer Veränderung des Zustands. Wenn das Teilchen aber vorher auch schon in dem Zustand war, dann nicht. Beispiel: Ein Elektron im Wasserstoff-Atom im Grundzustand. Wenn ich die Energie messe (wie auch immer), dann messe ich die des Grundzustands, und hinterher ist das Elektron im selben Zustand wie vorher. Das wird im zweiten Teil vermutlich deutlicher werden, wo ich etwas konkretere Beispiele angucke.

    Und danke für das Feedback – anscheinend ist es wirklich oft einfach nur die mathematische Horror-Hemmschwelle, die überwunden werden will.

    @Ludger
    Diese anschauliche Erklärung der Unschärfe ist ganz o.k., aber Unschärfe hatten wir ja noch gar nicht. So ganz glücklich bin ich mit der Unschärfe-Erklärung auch nicht.

    @Kallewirsch
    Was negative Temperaturen angeht, das ist eine kniffligkeit der Thermodynamik, in der man die Temperatur am besten über die Änderung der Entropie mit der Energie definiert. In manchen Systemen kann die Entropie abnehmen, wenn die Energie des Systems zunimmt, und da gibt es dann eine “negative” Temperatur. Vielleicht schreibe ich darüber irgendwann, es steht schon lange auf der Themenliste.

  56. #56 Ludger
    8. Januar 2013

    @ Martin: Deine Aussage

    Ich stelle mich hier deshalb erst einmal auf einen pragmatischen Standpunkt: Führt man eine Messung an einem Quantenobjekts durch, dann beeinflusst diese Messung den Zustand so, dass das Objekt hinterher in dem gemessenen Zustand ist. Klingt kompliziert? Nehmen wir wieder unser Beispiel von oben, ein Teilchen in einem Zustand

    hat also nichts mit der Unschärfe zu tun!? Meine Antwort bezog sich auf segeln141 – 7. Januar 2013 :

    ja,klingt kompliziert,wenn man die Aussage im Quantentalerartikel sich vor Augen hält.
    MarinB was ist die Lösung?

  57. #57 segeln141
    8. Januar 2013

    @Martin B

    Meist führt das zu einer Veränderung des Zustands. Wenn das Teilchen aber vorher auch schon in dem Zustand war, dann nicht. Beispiel: Ein Elektron im Wasserstoff-Atom im Grundzustand. Wenn ich die Energie messe (wie auch immer), dann messe ich die des Grundzustands, und hinterher ist das Elektron im selben Zustand wie vorher

    ganz klar ist mir das noch nicht.
    Denn Ludger schrieb zur “Erklärung”:

    Irgendwann kommt bei kleiner werdenden Quantenobjekten die Situation, dass die zur Messung eingesetzten Lichtquanten das Messobjekt bei der Messung “stoßen” und dabei dessen Geschwindigkeit oder Ort verändern

    Die Messung führt zu einer Veränderung des Zustandes,außer es war bereits in diesem Zustand.Wenn ich aber messe,warum wird dann dieser Zustand nicht wieder verändert?Warum sind beim ersten Mal die Lichtquanten in der Lage,den Zustand zu verändern,dann aber nicht mehr?

  58. #58 segeln141
    8. Januar 2013

    @MartinB

    wahrscheinlich hälst Du Deine Vorlesungen vor Studenten,die darauf “gefasst” sind und bereits mathematisch “vorgebildet” sind,sodass die Mathematik ihnen viel geläufiger ist.

  59. #59 Ulf Lorenz
    8. Januar 2013

    @segeln141:

    Am kuerzesten ist dazu die axiomatische Formulierung: Wenn du eine Messung machst, dann gehoert dazu immer ein Satz von Zustaenden und dazugehoerenden Messwerten (wir ignorieren mal den Punkt, wie man diese Zustaende fuer nichttriviale Faelle herausfindet).

    Wenn man die Energie misst, dann sind das stabile Energiezustaende mit den dazugehoerigen Energiewerten (z.B. die Orbitale im Wasserstoffatom und deren Energien). Wenn du den Ort misst, dann sind das Positionszustaende im Raum (“das Teilchen ist genau hier”) mit den dazugehoerigen Positionen.

    Bei der Messung bekommt man immer einen der moeglichen Messwerte und findet anschliessend das Teilchen im dazugehoerigen Zustand. Bekommt man also 1 Ry fuer die Bindungsenergie eines Elektrons im Wasserstoff, ist das Elektron nach der Messung immer im dazugehoerigen 1S-Orbital, selbst wenn der Zustand vorher noch Anteile in anderen Orbitalen hatte. Ludger hat also eine falsche Erklaerung, die allerdings sehr populaer ist (hatte ich auch als junger Student 🙂 ).

    Das hat sich halt so aus Experimenten ergeben. Warum das so ist, kann man auch konsistent innerhalb der Quantentheorie, aehm, motivieren (Dekohaerenz), allerdings kommt man irgendwann an eine metaphysische Barriere (Stichwort Wigners Freund), wo man mangels Experimente aufgeben und das einfach hinnehmen muss.

  60. #60 Ulf Lorenz
    8. Januar 2013

    Das bringt mich auch zu der Frage: Wie waere es eigentlich, die Quantentheorie direkt als Feldtheorie einzufuehren?

    Viele Probleme kennt man ja schon aus anderen Feldtheorien, z.B. dass ein Teilchen zugleich an mehreren Orten sein kann (wenn ich eine Antenne aufstelle und ein Funksignal aussende, dann befindet sich das dazugehoerige elektromagnetische Feld nicht an einem Ort, sondern ist “verschmiert”). Koennte das nicht helfen?

    Als Nachteil sehe ich, dass die Verbindung zur klassischen Mechanik und dazugehoerigen Begriffen (Energie, Impuls, Messung, …) erst recht spaet moeglich ist, aber das ist ja auch sonst etwas problematisch (zumindest wenn man es rigoros machen will).

  61. #61 MartinB
    8. Januar 2013

    @segeln
    “Warum sind beim ersten Mal die Lichtquanten in der Lage,den Zustand zu verändern,dann aber nicht mehr?”
    Das ist genau der Grund, warum die anschauliche Erklärung der Unschärfe nicht so ganz exakt ist. Im zweiten Teil wird es um Messgrößen gehen, die sich nicht gleichzeitig messen lassen, dann wird das vielleicht etwas klarer. Der oben verlinkte text zu Unschärfe und Nichtlokalität geht auch kurz auf das Problem ein.
    Es ist hier immer sehr schwierig, die Gratwanderung zwischen praktisch-anschaulich einerseits und theoretisch exakt andererseits richtig hinzubekommen.
    Meine Vorlesung halte ich übrigens vor MaschbauerInnen, denen mute ich nicht mal den Begriff der Wellenfunktion zu…

    @Ulf
    Die Quantentheorie als Feldtheorie hat dann aber den nachteil, dass die Wellenfunktion kein echtes Feld ist – es sei denn, du willst gleich Quantenfeldtheorie betreiben. Und dann wird’s auch wieder genauso schwierig, warum man im Doppelspalt einzelne Pulse sieht und eben keine kontinuierliche Energieübertragung.

  62. #62 mar o
    9. Januar 2013

    Schöner Artikel!

    Eine Anmerkung habe ich noch:
    Die Schreibweise |ψ> für einen Zustand mit Wellenfunktion ψ(x) ist nicht verzerrend, sondern mathematisch korrekt (Zusammenhang der beiden über die Vollständigkeitsrelation). Die Schreibweise |ψ(x)> hingegen ist falsch und trägt nur zur Begriffsverwirrung von Zustand und Wellenfunktion bei. Deswegen würde ich sie entweder rausnehmen oder klar machen, dass sie nicht gleichwertig mit der gängigen und korrekten Schreibweise |ψ> ist.

  63. #63 MartinB
    9. Januar 2013

    @mar o
    Hmm, ich bin da immer ein bisschen am Schleudern: ψ(x) wird ja auch gern als Bezeichnung für die Funktion verwendet (so wie viele Leute auch sin(x) schreiben, wenn sie die Sinus-Funktion meinen) – dann muss man mathematisch sauber zwischen der Funktion und ihrem Wert an einem beliebigen Punkt x trennen, aber das macht ja auch keiner wirklich (und viele Leute sagen umgekehrt: Ich betrachte ein Elektron mit WF ψ(x), was dann auch wieder nicht passt.)

  64. #64 WoSzym
    9. Januar 2013

    Wieder ein interessanter Artikel! Wie üblich hier, deutlich über das populärwissenschaftliche Maß hinaus und doch noch einigermaßen stemmbar für NichtphysikerInnen. Bezüglich der Akkzeptanz mathematischer Schreibweisen gegenüber der reinen “Anschaulichkeit” vermute ich, dass das auch zum großen Teil an der Gewöhnung (bei den Physikern) liegt, obwohl auch unsereins (Nichtphysiker) mathematische Schreibweisen akkzeptiert ohne sich diese vorstellen zu wollen. Unter 1 m^2 kann ich mir eine Fläche vorstellen, weiß aber auch, dass es das Produkt zweier Längen ist, 1 t^2 (zB a = 1/2 gt^2) benutze ich auch anstandslos als Produkt zweier Zeiten, ohne mir vorstellen zu wollen, was eine Quadratsekunde sein könnte. Von daher denke ich, daß es “Unvorstellbares” gibt, also nur mathematisch beschrieben werden kann.

  65. #65 mar o
    9. Januar 2013

    @MartinB:
    Das stimmt, da gibt es auch eine Verwechslungsgefahr. Eigentlich müsste man noch vorsichtiger formulieren. 😉

    Trotzdem finde ich die Schreibweise |ψ(x)> ziemlich irreführend. Kets |> sind basisunabhängige Elemente des Hilbertraus, x bezeichnet aber die Ortsbasis. Das passt nicht zusammen. |ψ> hingegen steht mit ψ(x) über ψ(x) = in einem wohldefinierten mathematischen Zusammenhang.

  66. #66 mar o
    9. Januar 2013

    Darstellungsprobleme, weil die Blogsoftware meinen Koeffizienten für ein html-tag hält.

    Dann so:
    ψ(x) = [x|ψ]

    wobei die eckigen Klammern eigentlich spitze sein sollen.

  67. #67 MartinB
    9. Januar 2013

    @mar o
    Ich finde das ket-psi(x) auch sehr irreführend – aber das ket-psi auch schon (ich spare mir mal das Rumärgern mit dem html-code-Mist…) – du hast recht, das ket-psi kann man sinnvoll interpretieren, aber irreführend sieht es für mich trotzdem aus.

  68. #68 Ulf Lorenz
    10. Januar 2013

    @Martin

    Ich dachte eher daran, dass, wenn man psi(x) als Ein-Teilchen-Feld auffasst, sich viele Sachen wie z.B. der Doppelspalt oder die Unschaerferelation auf bekannte Gegenstaende zurueckfuehren lassen (hier: Elektrodynamik). Allerdings hast du recht, man muss in der Tat noch die diskreten Teilchenzahlen einbauen,
    daran hatte ich nicht gedacht.

    Eventuell faellt das dem Leser erst auf einem hoeheren Niveau auf, aber das muesste man wohl ausprobieren, und ich habe hier keine guten Testobjekte um mich herum…

  69. #69 AJOM
    10. Januar 2013

    @MartinB
    Vielen Dank für die ganze Mühe mit diesem (und anderen) Artikeln.
    Als Informatiker habe ich mit Formeln kein Problem, sie sind im Gegenteil oft prägnanter.
    Trotzdem zwei Frage:
    1) Die psi(x)^2 beschreiben die Wahrscheinlichkeit (Wk) für Zustand x. Angenommen es gibt unendlich viele x (z.B. stetige Orte) und unendlich viele davon haben eine positive Wk. Wie können die Wken (die sich ja zu ein addieren müssen) dann für unendlich viele x positiv sein?
    2) Mir wurde noch nicht klar, wieso die psi(x) genau einer Wellenfunktion folgen müssen; diese kam etwas unvermittelt ins Spiel. Was ist die Motivation dafür? (soweit ich weiß kann man es nicht streng begründen, sondern nur motivieren).

  70. #70 MartinB
    10. Januar 2013

    @AJOM
    Die erste Frage habe ich oben im Warnschild ganz kurz angesprochen: In dem Fall ist dann ψ(x) eine Wahrscheinlichkeitsdichte. Die Wahrscheinlichkeit, das teilchen exakt an einem Ort u finden, ist dann Null, stattdessen fragt man nur nach der Wahrscheinlichkeit, das teilchen in einem kleinen Raumvolumen zu finden, die ist dann gegeben durch das Integral über ψ(x) dV.

    Bei 2 weiß ich nicht genau, was du meinst, ψ ist die Wellenfunktion – wer muss da einer WF folgen?

  71. #71 AJOM
    10. Januar 2013

    zu 2). ich meine damit, wieso phi(x) genau die Wellenfunktion (und nicht irgendeine andere Funktion) ist, Dabei interpretiere ich phi erstmal allgemein als Symbol für eine beliebige Funktion und assoziiere damit nicht gleich die Wellenfunktion. Angenommen ist wüßte nicht, welche Funktion die einzelnen Zustände gewichtet, was ist die Motivation dafür die Wellenfunktion zu wählen?

  72. #72 MartinB
    10. Januar 2013

    @AJOM
    Die Wellenfunktion ψ(x) ist genau dadurch definiert, dass sie den Wert der Wahrscheinlichkeitsamplitude am Ort x angibt – da gibt es keine Wahlfreiheit, irgendeine andere Funktion zu wählen. Vermutlich verstehe ich immer noch nicht, was du eigentlich meinst?

  73. #73 AJOM
    10. Januar 2013

    ok, dann versuche ich es so:
    wieso ist es sinnvoll, als Funktion, die die Wahrscheinlichkeitsamplitude am Ort x angibt, die Wellenfunktion zu nehmen bzw. diese als die Wellenfunktion zu definieren? Diese Definition muss doch irgendwie motiviert werden können, ansonsten könnten doch auch andere Funktionen entsprechend definiert werden, den Wert der Wk-Amplitude am Ort x anzugeben. Die Wellenfunktion hat sich wohl bewährt, aber was veranlaßt dazu anzunehmen, dass Wellenfunktion die Wk-Amplituden besonders gut widergibt (besser als irgendeine andere Funktion).
    Hoffentlich wird die Frage nun klarer.

  74. #74 MartinB
    10. Januar 2013

    @AJOM
    Ich verstehe es immer noch nicht. Es gibt eine Funktion, die die Wahrscheinlichkeitsamplitude am Ort x angibt. Diese Funktion bekommt als namen den Namen “Wellenfunktion”. Es gibt nicht eine “Wellenfunktion” und die ordnet man dann der Amplitude zu.

    “Wellenfunktion” ist nicht der Name einer bestimmten mathematischen Funktion (wie “sinusfunktion”), sondern ist einfach der Name der ortsanhängigen Wahrscheinlichkeitsamplitude.

  75. #75 AJOM
    10. Januar 2013

    ok, jetzt wird’s mir klarer. Die Wellenfunktion erhält man als Lösung der Schrödingergleichung. Diese wird wahrscheinlich in der entprechenden Artikelserie motiviert.

  76. #76 MartinB
    11. Januar 2013

    @AJOM
    Genau. Die Wellenfunktion ist die Wahrscheinlichkeitsamplitude als Funktion des Ortes. Man bekommt sie als Lösung der SGL.

  77. […] sind zwei Teile erschienen, hier und […]

  78. #78 MartinB
    13. Januar 2013

    Nachtrag: Ich habe jetzt den Nichtlokalitäts-Artikel wieder geflickt (wie gut, dass ich mir vor dem Server-Umzug eine lokale Kopie aller Artikel gezogen hatte…)
    https://scienceblogs.de/hier-wohnen-drachen/2011/05/29/quantenmechanik-nichtlokalitat-und-unscharfe/

  79. #79 Achim
    25. Januar 2013

    Toller Artikel, den ich erst jetzt entdeckt habe.

    Als Laie bin mir allerdings nicht ganz sicher, ob ich den Zusammenhang zwischen ψ(x) und dem Beispiel

    |Zustand> = ψ(x1) |x1> + ψ(x2) |x2>+ ψ(x3) |x3> + …

    richtig verstanden habe, oder anders ausgedrückt, zwischen “gesamter Zustand” und Wahrscheinlichkeitsamplitude.

    Gilt denn |Zustand> = |ψ(x)> ?? Oder ist das falsch?

    Gruß Achim

  80. #80 MartinB
    25. Januar 2013

    @Achim
    Das kann man so sagen, auch wenn es ein bisschen problematisch ist (siehe die kommentare oben). Wichtig ist am Ende nur, dass man den Zustand eindeutig charakterisiert.

  81. […] brauchen wir wieder einmal die Grundlagen der Quantenmechanik. Da ich die neulich ausführlich erklärt habe, mache ich es hier kurz (und ein bisschen schlampig): Quantenmechanische Systeme können sich in […]

  82. #82 blödmannsgehilfe
    Kanaren
    2. April 2013

    @ MB
    ich bin über deinen neuesten (#81) Artikel hierher gekommen. Ich finds gut, dass du nur “hier” und ” da” unterscheidest – das macht das fies komplexe Thema übersichtlich – und die Erweiterung “dort” sinnvoll. Und wer denken kann, weiss, es existiert noch “quasi alles dazwischen”. Auch die Formeln finde ich gut, wer sie nicht mag, kann getrost drüber weglesen. man kann es eh nicht allen recht machen…
    Letztlich hat mich der Artikel begeistert – hat er mich weitergebracht? Logisch ja, intuitiv nein. Ich tue mich einfach noch schwerer, den SINN dahinter zu erahnen und frage michnicht zuletzt, was können wir (überhaupt) (er)denken/ -kennen/ -ahnen. Während des Studiums habe ich mich vorwiegend mit Wittgenstein und Husserl herumgeschlagen und musste Jahre danach einsehen – die wussten (letztendlich) auch nicht mehr (als ich).
    Die (klassische) Physik ist ja irgendwie noch (zumal durch unsere (Alltags)Erfahrungen) nachvollziehbar – aber Quantenmechanik. Und wenn sie uns schon wirr vorkommt – wer sagt unsnicht, dass sie ttsächlich wirr (quasi unreal) ist und uns unser Verstandnur (Irgendwas) vorgaukelt?
    (Logisch) kann ich die (klassische) Astrophysik irgendwie nachvollziehen, wenn mir haarklein erklärt wird, wie die Welt bis kurz vor dem Urknall (Planck-Zeit) ausgesehen hat. Aber die gleiche Physik, die mir die Zustände bis quasi zum Urknall plausibel erklärt, hat keine Idee, wie/ woraus 95% des Universums besteht (Dunkles Zeugs halt, wie “Materie” … “Energie”….)
    Klar, die Urknall-Theorie ist extrem plausibel – aberwenn ich jemandem erzähle: “Alter ich weiss absolut bescheid – zu 5%” – dann werde ich nicht wirklich überzeugen…
    Letztlich wieder auf deine Artikel zurück: da stellt sich zutiefst die Frage: was KÖNNEN wir erkennen, was LÄSST die Welt überhaupt zu, das wir zu erkennen glauben?

    Logisch mag ich sowohl Astrophysik als auch Quantenmechanik, weil sie unsere Welt irgendwie (ein)ordnen – aber intuitiv (oder – wissenschaftlich betrachtet: philosophisch) halte ich das (alles) für kompletten Unsinn…

  83. #83 blödmannsgehilfe
    Antarktis
    2. April 2013

    Besonders gemocht habe ich diesen Artikel, weil er mich zunächst an eine Situation aus der Sesamstrasse erinnert hat: Ein kleiner Junge kommt zu Krümelmonster und dieser sagt: “Hey Kleiner, was machst du denn hier?” … der sagt: “Hier? Ich will aber DA sein!!” … “DA ist Grobi.” … der ein paar Meter weiter weg steht. Der Kleine läuft zu Grobi: “Hey Kleiner, was machst du denn hier?” “HIER?!? Ich will aber DA sein” “NA, DA ist Krümelmonster….” Der Kleine rennt zurück zu Krümelmonster: “Hey Kleiner, was machst du denn HIER?” “HIER?!? Ich will aber DA sein!!!!!!”
    Naja, das geht noch ne Weile so, bis es beiden Monstern zu dumm wird, sie gehen und sagen “Hey, Kleiner, du bleibst aber DA!” ….endlich ist er “da”…
    Was sagt uns dieses Quasi-Gleichnis über die Quantenmechanik resp. unsere Rezeption unserer (Um)Welt?

    Vielleicht sind wir die “Monster” und die Quantenteilchen der kleine Junge, der in unserem Fall nicht “da” sondern “hier” sein will, als bei uns, also beobachtet werden WILL?
    Wenn das Teilchen erst einmal beobachtet worden ist, dann ist es auf jeden Fall “HIER”… wo es beobachtet wurde….

  84. #84 Wargomar
    9. Juli 2013

    Moin.
    Keine Ahnung, wie man versuchen kann die Quantenphysik, für Laien wie mich, “verstehbar” zu machen.
    Physiker, die sich mit dieser Disziplin eingelassen haben, erklären die Qauntenphysik als “unverstehbar”.
    Man weiss zwar, daß es funktioniert, aber bei den meisten Prozessen weiß niemand warum und wie.
    Der meschliche Vestand hat eben seine Grenzen. Gott sei Dank. Sonst würden wir wahrscheinlich neue Massevernichtugswaffen entwickeln.

  85. #85 MartinB
    9. Juli 2013

    @Wargomar
    “Physiker, die sich mit dieser Disziplin eingelassen haben, erklären die Qauntenphysik als “unverstehbar”.”
    Jein – deswegen schreibe ich ja den Artikel.

    “Der meschliche Vestand hat eben seine Grenzen. Gott sei Dank. Sonst würden wir wahrscheinlich neue Massevernichtugswaffen entwickeln.”
    Genauso könnte ich sagen “Der menschliche Verstand hat Grenzen. Schade. Sonst hätten wir längst ein Heilmittel gegen Krebs.”
    Ist mir in beiden Richtungen zu simpel.

  86. […] Der Physiker Martin Bäker hat in seinem Blog “Hier wohnen Drachen” eine schöne Einführung […]

  87. #87 NorbertA
    Schopfheim (Baden Württemberg)
    18. Juli 2013

    Okay, ich bin spät, aber ich bin froh, diesen Blog gefunden zu haben (eben gerade über den Link von aargks) und habe nun meine Sommerbeschäftigung gefunden, galube ich.

    Mein Grundproblem mit der Quantenmechanik: Mir ist klar, dass man ein mathematisches Weltbild aufbauen kann, in dem man Gelichungen aufstellt, diese löst und dann irgendetwas weiss. Schön. Mir fehlt aber der Bezug zur Realität, da kriege ich die Kurve nicht. Ich bin Ingenieur von Beruf und habe von daher einige Erfahrungen mit dem Messen kleiner Größen, allerdings im industriellen Maßstab. An 0,001 mm haben wir uns ganz schön die Zähne ausgebissen. Daher komme ich einfach nicht über diese ‘Größenschranke’. Dieses hängt in meinem Unterbewusstsein und blockiert irgendwie den Verstand. Vielleicht etwas klarer:

    In einem Artikel im Skeptiker hat Philippe Leick vor einiger Zeit über den Nachweis der Verschränkung berichtet, der mit Hilfe von verschränkten Photonen durchgeführt wurde. Dies bedingt, wenn ich ein bischen rechne, die Messung unheimlich kleiner Energien und unheimlich kleiner Zeitunterschiede. Da taucht dann die bisher für mich ungelöste Frage auf: Wie misst man denn das? Mit welchem Instrument kann ich die Energie eines einzelnen Photons indentifizieren und wie messe ich Zeiten, die sich aus der Lichtgeschwindigkeit und ein paar Zentimeter Abstand ergeben?

    Ich weiß zwar nicht, ob ich am Ende tatsächlich etwas von Quantenmechanik verstanden haben werde, aber über diese Eingangshürde komme ich einfach nicht drüber hinweg.

  88. #88 MartinB
    21. Juli 2013

    Bin im Urlaub, deshalb nur kurz:
    Einzelne Photonen kann man z.B. mit einem Photomuiltiplier nacweisen.
    Ansinsten misst man z.B. Energieniveaus von Atomen mit den Spektrallinien.
    Die Zeiten, wo man sich an nem Mikrometer die Zähne ausgebissen hat, sind vorbei – heute kann man mit nem Elektronenmikriskop problemlos einige zehn Nanometer auflösen, mit einem Rasterkraftmikroskop sogar noch mehr.
    Die experimentelle Seite kommt bei mir im Blog meist zu kurz – erstens bin ich da kein Experte, zweitens interessiert mich das nicht so. Aber hier gibt es irgendwo auf dem Blog (ich glaube, letzten Sommer war’s) ein Video, wo die Quantenverschränkung direkt sichtbar gemacht wurde.

  89. #89 NorbertA
    4. August 2013

    Danke erst einmal (auch ich bin praktisch in Urlaub, daher die späte Reaktion.

  90. #90 NorbertA
    4. August 2013

    Danke erst einmal (auch ich bin praktisch in Urlaub, daher die späte Reaktion.

    Irgendwie glaubt die Kommentarfunktion hier, ich hätte dies schon einmal gesagt – habe ich meines Wissens aber nicht. Nun füge ich diesen Text hier hinzu, damit des inhaltlich anders aussieht. Der Moderator möge bitte den letzten Absatz streichen.

  91. #91 NorbertA
    4. August 2013

    @admin:
    Dann war’s doch doppelt. Man nehme bitte diesen und den letzten Kommentar weg.

    Tschuldschung.
    Norbert

  92. […] in der Quantentheorie haben Felder nämlich meist keine eindeutigen Werte, sondern befinden sich in Überlagerungszuständen. (Der Link erklärt dasmit der Überlagerung ausführlich, hier die […]

  93. […] Das habe ich auch im oben verlinkten Artikel über Feynmandiagramme, aber auch hier […]

  94. #94 Bert
    Ruhrgebiet
    29. Dezember 2013

    Ich begebe mich seit neuestem in den Versuch “Quantenmechanik zu verstehen” – vor längerer Zeit mal angestoßen durch “Skurrile Quantenwelt” u. a. Ich begann zu lesen mit dieser Seite vom 5.1.2013 und es gefällt mir hervorragend.
    Frage:
    Ist es sinnvoll beim Einarbeiten eine bestimmte Reihenfolge zu berücksichtigen?
    mit begeisterten Grüßen

  95. #95 MartinB
    29. Dezember 2013

    @Bert
    Diese Serie hat drei Teile, die bauen aufeinander auf, die solltest du also in der richtigen Reihenfolge lesen. Vermutlich ist es hilfreich, außerdem die texte “QM und Realität” und “die beliebtesten Phrasen…” (rechts unter Artikelserien zu finden) zu lesen – und dann vielleicht diese drei Teile nochmal anzugucken. Und aktuell schreibe ich ja gerade an einer kleinen Serie “Ein Teilchen fliegt von A nach B2, da geht es im 2. Teil (und in einigen, die noch kommen) auch um QM, das beleuchtet alles nochmal aus nem anderen Blickwinkel.

    Meiner Ansicht nach lernt man Physik am besten, wenn man dieselbe Sache immer wieder aus anderen Blickwinkeln anguckt, deswegen gibt es auch ziemlich viele Texte zur QM (die Suchmaschine oder ein Klick auf die tags wirft noch einige weitere aus)-

    Generell ist es auf dem Blog aber so, dass texte, die nicht explizit als Serie deklariert sind, in beliebiger Reihenfolge gelesen werden können (deswegen wiederholt sich auch vieles).

    Und bei Fragen immer einfach nen Kommentar hinterlassen.

  96. […] Wie beschreiben wir den Zustand mit festem Impuls im Pfadintegral-Bild? Das ist auf den ersten Blick nicht so einfach, denn wir sind ja immer an einem Ort (A) losgelaufen und an einem anderen Ort (B) geendet – jetzt aber wissen wir den Ort nicht, sondern kennen nur den Impuls am Anfang und am Ende. Wir können aber nach den Regeln der QM den Zustand mit festem Impuls am Anfang als eine Überlagerung aus lauter Zuständen ansehen, bei denen das Teilchen an einem bestimmten Ort ist. (Das mit den Überlagerungen habe ich in meiner QM-Serie ausführlicher erklärt.) […]

  97. #97 Phillip
    Magdeburg
    25. Februar 2014

    Lieber Martin,

    für einen interessierten Laien ist dein Blog wirklich ein Segen!
    Vielen vielen Dank!!

    Liebe Grüße,
    Phillip

  98. #98 MartinB
    25. Februar 2014

    @Phillip
    Danke für’s Lob 🙂

  99. #99 gant
    28. Februar 2014

    Jemandem etwas zu erklären, das er nicht versteht, wird nicht unbedingt dadurch erleichtert, dass man alles noch komplizierter, ausführlicher oder abstrakter darstellt, sondern zum Beispiel auf einen Sachverhalt zurückführt, der dem Fragenden bekannt ist.

    Sie jonglieren offensichtlich ganz gut mit Formeln, ob Sie deshalb eine bessere Ahnung von der Interpretation Ihrer Beobachtungen haben …. bezweifel ich

    Ich bin kein Physiker, ich beziehe hier meine Kenntnisse nur aus populärwissenschaftlicher Lektüre. Ich kann aber stringente und logische Argumentation erkennen.

  100. #100 MartinB
    28. Februar 2014

    @gant
    Erklärungen ohne Formeln gibt es hier im Blog ja zu Hauf (siehe rechts unter Artikelserien). Hier in diesem Artikel geht es ja genau darum ,die Brücke zwischen populärwissenschaftlichen und wissenschaftlich sauberen Argumenten zu schlagen. Zumindest bei einigen LeserInnen scheint das ja auch gut angekommen zu sein, wie die Kommentare belegen.

  101. #101 Horst O. Koch
    Altensteig
    6. Oktober 2014

    Vielen Dank für den tollen Blog – ich sauge die Informationen regelrecht auf.
    Und ich freue mich über jeden Exkurs in die Mathematik – allerdings habe ich ein Sprachproblem.
    Seit dem Abi musste ich keine mathematischen Aussagen mehr in Worte fassen und jedes Mal wenn so ein Ausdruck dasteht, ringe ich um Worte. Ich habe zwar genug Phantasie, mich verbalkreativ zu betätigen, bin aber unzufrieden damit.
    Gibt es irgendeinen Weg, auf dem ich erfahren kann, wie der Fachmann die Ausdrücke in Worte fasst?

  102. #102 MartinB
    6. Oktober 2014

    @Horst
    Weiß nicht, ob es so einen Übersetzer gibt. Bei vielen Dingen ist es auch individuell verschieden, z.B. ob man
    “a – hier” oder
    “a mal hier” oder
    “a ket-hier” (weil die Zustandsklammern “kets” heißen) sagt.
    Wenn du konkrete Fragen hast, immer her damit.

  103. #103 Schaf
    20. Januar 2015

    WOW, ich habe die Quantenmechanik immer gehasst, weil ich nie etwas verstanden habe. Ich habe alle Vorlesungen besucht und alle Übungen mehrmals mitgemacht und konnte keinen Satz lang folgen. Nun muss ich aber wohl oder übel meine Prüfung absolvieren und bin zufällig auf diese Seite gekommen. Einfach der Wahnsinn – endlich fange ich mal an etwas zu verstehen!! Vielen vielen Dank für diese tolle Erklärung!! Mit solchen Worten müsste eine Vorlesung beginnen und nicht gleich mit den Unterschieden der zeitabhängigen und -unabhängigen Schrödingergleichung …. 🙂
    Ich bin einfach sprachlos – klasse :):):)

  104. #104 MartinB
    20. Januar 2015

    @Schaf
    Danke für das Lob. Die Unterschiede zwischen der zeitunabhängigen und der zeitabhängigen SGL findest du übrigens in meiner SGL-Serie erklärt. Einfach rechts bei Artikelserien klicken. Dort findest du auch den Link auf den 2. und 3. teil dieser Serie (und noch ne Menge mehr…)

    Und gern die Seite an alle KommilitonInnen weiterempfehlen.

  105. #105 Martina
    München
    25. Januar 2015

    Vielen herzlichen Dank!!!
    Mein Lehrer (9. Klasse Realschule) erwartet von mir dieses Thema zu erklären, ich habe echt sehr sehr lange gesucht bis ich das gefunden hab etwas das ich zum Großteil verstehe, und ich danke dir dafür das du dir so viel Zeit genommen hast um das zu schreiben, außerdem hat es eine 9. Klässler zum Großteil verstanden. Danke

  106. #106 MartinB
    25. Januar 2015

    @Martina
    Wow, für ne 9. Klasse ist das aber sehr heftig. Normalerweise studiert man dafür ein paar Semester Physik…
    Ich drücke jedenfalls alle Daumen (die Serie geht übrigens noch weiter, siehe rechts bei Artikelserien). Vielleicht helfen auch die beiden einführenden Artikel über “die beliebtesten Phrasen” und “Qm und Realität”, um das Bild abzurunden.

  107. #107 robert th.
    Wedel
    29. März 2015

    lieber Martin, es ist richtig, dass die formulen kurzen die erklärung, nur: wieviel leute in die welt verstehen sie!? Die shah-spiel erklärt man schell mit F3 u.s.w., aber die meisten leute brauchen die wörtlichen erklärung über die bewegung der figuren, egal wie mühsam das ist. Die natur braucht nicht die formulen und ekuacionen, diese brauchen die natür. Also, wenn die QM die natür erklären will (o. soll), dann muss sie einfache erklärungen geben, so einfach, dass man jeder mensch auf der welt versteht. Sonst ist sie (besonders mit solcher menschlicher conditionanzen, wie Schröders gleichungen, etc.) nur eine sofismus um die natür zu erklären. Von duzend artikel, die ich über QM gelesen habe, deine sieht es etwas besser, aber noch nicht genug klar für jeden mensch, um zu verstehen: was ist eigentlich QM und wie sie die natür-funktion erklärt. Es gibt über 30 konstanten von der “wissenschafter”-herde, die die natür erklären sollen!!! Braucht die natür all die 30 konsonanten, oder haben wir hier zu tun mit ekletismus und idiotizmus!?!

  108. […] mal eine kleine Erinnerung an eine der Grundregeln der Quantenmechanik (ausführlich habe ich das hier erklärt): Anders als in der klassischen Physik können quantenmechanische Systeme in einem sogenannten […]

  109. #109 Liancaley
    17. August 2015

    Klasse Artikel, aber
    Klugscheißermode an
    bei dem Vergleich mit den Lottowarscheinlichkeiten:
    3 richtige hat die Warsch. 10 %
    4 richtige hat die Warsch. 1%
    Dann hat das Ereignis 3 oder 4 richtige nicht die Warsch. 11%, da das Ereignis mit den 4 richtigen doch auch das Ereignis mit den 3 richtigen miteinschließt(oder andersrum). Ich bin mir nicht ganz sicher, rechne lieber nach, falls ich falsch liege tut es mit Leid
    Klugscheißermode off

  110. #110 MartinB
    18. August 2015

    @Liancaley
    Gemeint ist natürlich “genau 3″ bzw.”genau 4” – denn das ist ja das, was den Gewinn, den ich bekomme, bestimmt. Niemand. der 6 Richtige im Lotto hat, sagt “ich hatte 6 Rictige und 5 richtige und 4 richtige usw”, und wenn ich 4 Richtige habe, bekomme ich auch nur den Gewinn für 4 Richtige ausbezahlt und nicht zusätzlich noch den für 3 richtige,

  111. #111 Liancaley
    20. August 2015

    @MartinB
    Gut das macht Sinn, da hab ich nicht ganz nachgedacht.
    Schade eigentlich dass man nur den Gewinn für die größte Übereinstimmung erhält und nicht für alle 😀

  112. #112 MartinB
    20. August 2015

    @Liancaley
    Ja, aber da die ja jeweils ume twa ne Zehnerpotenz abnehmen, kommtes nicht wirklich drauf an…

  113. #113 Liancaley
    20. August 2015

    Bin eh noch minderjährig, darf daher eh nicht spielen…

  114. #114 MartinB
    20. August 2015

    @Liancaley
    🙁

  115. #115 Liancaley
    20. August 2015

    Hab ich kein Problem mit, gibt sinnvollere Wege sein Geld zu nutzen…

  116. #116 Dani L.
    München
    14. September 2015

    Jetzt ist mir auch ein Licht aufgegangen. Ich finde es sehr gut gemacht.

  117. #117 MartinB
    15. September 2015

    Freut mich.
    Es gibt auch noch Teil 2 und 3 (Links rechts unter Artikelserien).

  118. #118 Kris
    Giessen
    19. September 2015

    Sehr gute Erklärung , die Mitdenk-Aufgaben waren am Besten , weil man da direkt in die Erklärung mit einbezogen wurde.

  119. #119 MartinB
    19. September 2015

    Danke für’s Lob, sollte ich vielleicht öfters so machen.

  120. #120 Peter Wiedemann
    Neuburg an der Donau
    15. März 2016

    Vielen Dank für den Artikel. Der hilft beim Einsteigen in die Thematik gewaltig, weil nun doch die mathematischen Zusammenhänge geschildert werden, ohnen einen gleich zu erschlagen. Wie spricht man noch dieses Wahrscheinlichkeitsamplitudenzeichen aus?

  121. #121 MartinB
    15. März 2016

    @Peter
    Meinst du das ding, das aussieht wie Poseidons Dreizack? Das ist ein “psi”, hat aber nix mit übersinnlichen Kräften zu tun.
    Wenn du rechts bei den Artikelserien klickst, findest du übrigens auch noch eine zur Schrödinegrgleichung, die hilft vielleicht auch beim verstehen.

  122. #122 Fiona
    Hier
    14. April 2016

    Ich finde es Super geschrieben echt hilfreich. Auch ohne Formeln und Mathematik Kentnisse verständlich. Ich hoffe wenn wir das Thema irgendwann in der Schule machen Errinert ich mich an diesen Beitrag. And ich Martin B gelesen habe musste ich an meinen ehemaligen Physiklehrer denken er heiß Martin Becker

  123. […] aller Bemühungen, Dinge wie die Quantenmechanik anschaulich zu erklären, ist das Ganze schon ziemlich abstrakt. Oder besser gesagt: abstrakt gewesen. Denn zumindest einen […]

  124. #124 MartinB
    14. April 2016

    @Fiona
    Freut mich sehr.

  125. #125 Franz
    21. Juli 2016

    https://scienceblogs.de/hier-wohnen-drachen/2013/01/05/quantenzustaende/4/#comment-25298

    Wo jetzt, links oder rechts?
    Spaß beiseite… schöner Artikel, danke sehr! Freu mich schon, auf deiner Seite weiter zu schmökern. Allerdings wäre es für die Serien hilfreich, wenn du am Ende von Teil X einen Link auf Teil X+1 setzen könntest, würde das Weiterlesen sehr erleichtern.

  126. #126 Fritz
    21. Juli 2016

    Der Einfachheit halber stelle ich mir ein Elektron in einem Orbital vor. Nach dem was ich jetzt alles über Quantenmechanik gelesen habe, gibt es für mich 2 Möglichkeiten mir eine Elektron vorzustellen:

    1. Das Elektron verhält sich wie eine wild gewordene Biene, die mit rasender Geschwindigkeit in ihrem Orbital herumsaust, und an manchen Orten häufiger anzutreffen ist als an anderen. Da sie so unglaublich schnell ist, sieht man von außen nur eine Wolke, die an manchen Stellen dunkler ist als an anderen Orten. Führt man nun eine Messung durch, beschießt man das Orbital mit einem Vorhang aus Kugeln, die alle so schwer sind wie die Biene selbst. An einer bestimmten Stelle und zu einem bestimmten Zeitpunkt trifft dann die Biene mit einer Kugel (k1) zusammen. Dadurch wird diese Kugel (k1) abgelenkt und die Biene ebenfalls. Die Messung hat also den Ort der Biene beeinflusst.

    oder

    2. Bei einem Elektron handelt es sich nicht um ein sehr kleines begrenztes sondern um ein räumliches Objekt, das von einer Form in eine andere Form fließen kann, ähnlich einer Amöbe die keine feste Gestalt hat. Schaut man nun von außen auf die Amöbe, ist sie an manchen Stellen dicker und an machen etwas dünner.

    Ich persönlich favorisiere bisher ja die erste Vorstellung. In diesem Fall wäre es jedoch kein Widerspruch zur klassischen Physik. Denn das Elektron befindet sich ja zu einem Zeitpunkt an einem bestimmten Ort. Nur weil es halt so unglaublich schnell ist, sehen wir von außen nur eine Wolke. Und weil ein Elektron so unglaublich klein ist, ist jede Messung so grob, dass sie automatisch auch den Ort des Elektrons beeinflusst. Denn wenn ich ein Elektron (x) mit anderen Elektronen (y) beschieße, dann ist das Werkzeug (y) ja genauso groß wie das Untersuchungsobjekt (x) selbst.
    Um jedoch mit dem Aufenthaltsort eine Elektrons rechnen zu können, ist es notwendig mit Wahrscheinlichkeiten zu rechnen, also ein Elektron ist mit einer bestimmten W’t an einem bestimmten Ort (x,y,z). Aber halt nur weil wir es nicht genau wissen. Faktisch kann das Elektron ja nur zu einem Zeitpunkt t1 an einem bestimmten Ort (x1,y1,z1) sein. Nur solange wir es nicht genau wissen, können wir für jeden möglich Ort nur eine bestimmte W’t angeben. Ist das richtig so?

    Vielen Dank für deine Antwort!

  127. #127 Alderamin
    22. Juli 2016

    @Fritz

    Weder noch. Man kann einzelne Elektronen der Reihe nach durch einen Doppelspalt schießen und erhält ein Interferenzmuster auf einem Leuchtschirm hinter dem Spalt. Schließt man jeweils einen der Spalte oder misst man auch nur, welchen Spalt jedes Elektron jeweils nimmt, ist das Interferenzmuster sofort weg. Daraus kann nur folgen: Das Elektron geht durch beide Spalte! In der Quantenwelt sind die Objekte, solange sie nicht interagieren und so “Stellung beziehen müssen” nicht an einem bestimmten Ort verortet, sondern man kann sie mit abnehmender Wahrscheinlichkeit auch in größerer Entfernung vom erwarteten Ort aufspüren.

    Das Elektron ist also im Orbital nirgendwo an bestimmter Stelle. Das Orbital beschreibt nur eine Region, wo sich das Elektron mit hoher Wahrscheinlichkeit wiederfände, sobald man es dort messen würde. Mit sehr geringer Wahrscheinlichkeit würdest Du es sogar auf dem Mond finden können.

    Versucht man übrigens den Radius eines Elektrons zu messen, erhält man das Ergebnis: im Rahmen der Messgenauigkeit punktförmig. Also keine Amöbe.

  128. #128 Fritz
    23. Juli 2016

    Hallo Alderamin,

    vielen Dank für deine sehr hilfreiche Antwort.
    Das Doppelspaltexperiment offenbart wirklich ein Paradox. Wenn man ein Elektron oder Photon misst, ist es punktförmig, also wie eine Teilchen und wenn man es durch einen Doppelspalt schießt, geht es durch beide Spalte gleichzeitig.
    Jedenfalls mein Horizont reicht nicht dafür aus, mir vorzustellen, wie ein Elektron beschaffen ist, und wie sich Materie wie eine Welle verhalten kann. Das ist absolut verrückt! Und die Verschränkung ebenfalls..

  129. #129 Alderamin
    23. Juli 2016

    @Fritz

    Das geht nicht nur Dir so. Die Welt ist im Kleinen nicht so, wie wir sie gewohnt sind, deswegen können wir sie nicht mit Modellen aus dem Makroskopischen beschreiben. So etwas wie ein konkreter Ort oder eine konkrete Geschwindigkeit gibt es in der Mikrowelt nicht (und wenn, dann nicht in Kombination). Teilchen sind auch keine kleinen Kugeln. Man kann sie nur durch die Summe ihrer Eigenschaften und anhand der physikalischen Gesetze beschreiben, die man durch Experimente gefunden hat. Jede Veranschaulichung kann nur Teilaspekte davon erfassen.

    Am ehesten noch kann man sie als Anregungen von Feldern betrachten, so wie ein Wellenpaket, das als Druckwelle durch ein Material oder ein Gas läuft, aber ohne auseinander zu laufen oder sich bewegen zu müssen (Schallwellen können zwischen zwei reflektierenden Wänden auch stehen bleiben). Aber Verschränkung ist damit auch nicht nachzuvollziehen. Eben immer nur Teilaspekte.

  130. #130 Fritz
    23. Juli 2016

    @Alderamin

    “Man kann sie nur durch die Summe ihrer Eigenschaften und anhand der physikalischen Gesetze beschreiben, die man durch Experimente gefunden hat.” Diesen Satz finde ich extrem erhellend. Eine andere Chance sehe ich momentan nicht, eine Vorstellung von der Quantenwelt zu bekommen. Danke nochmals!

  131. #131 Patrick
    Augsburg
    27. Juli 2016

    Sehr geehrter Herr Bäker,

    Ich habe ihren Artikel mit größter Aufmerksamkeit verfolgt und ausnahmsweise mal recht viel verstanden was sie im Artikel aufgezeigt haben (ich hatte mich schon ein einiger Fachliteratur versucht, aber da war ich halb ausgestiegen.) Um ihnen ein Profil von mir zu geben (da sie vorallem nach Feedback von Nichtphysikern gefragt haben) Ich bin gerade fertig gewordener Abiturent der im “Gammel 8” Physik bereits in der 10ten Klasse abgewählt hat. Dies begründete sich nicht an dem Fach Physik selber sondern an meiner Lernfaulheit. Ich fand dann größtes Interesse am Fach Chemie, welches ich nun auch studieren möchte. Ich habe explizit nach dem Thema Quantenmechanik/ Quantentheorie geforscht, da ich finde das Quanten (oder das Verständnis von ihnen) ein grundlegenderes Gerüst für unsere welt als das Borsche Atommodel sein könnten. (Ich bin noch zu unwissend um diese Aussage zu bejahen oder verneinen, deswegen Konjunktiv). Ebenfalls mein Interesse geweckt hat die neuste Entwicklungsankündigung von Google, dass sie mithilfe eines Proffessors den Quantencomputer entwicken wollen. (https://www.heise.de/newsticker/meldung/Google-In-zwei-Jahren-zum-Quantencomputer-3120283.html)
    (Als nächstes möchte ich als Laie darauf hinweisen das ich durchaus etwas missverstanden haben könnte!)

    Zuerst einmal möchte ich allen Vorschreibern die ihnen bei der erklärung der Tatsache Quadrate und Aplituden wiedersprechen die meinten ihre erklärungen sein unverständlich. Sofern ich verstanden habe hat das Quant verschiedene Zustände. (Da, Dort, hier) Und Zwischenzustände, die Fließend verlaufen. (Beim lesen bitte die Zeichnung von Seite 2 zurate ziehn)
    Der Quantenzustand ist ja anhand einen Kreises verbildlicht (Und aus diesem Leitet sich die Wahrscheinlichkeitsamplitude ab) Die Vorleser betrachten dabei nur die Entfernungen des Quantens (Welcher zwischen A und B wechselt) zu ihrem Festgelegten Zustand (A oder B). Was aber Zählt ist die Entfernung zum “Nullpunkt” des Systems (Koordinanten Ursprung) Die Entfernung von diesem Mittelpunkt bis zum Kreißäußeren beziffert die Quantenenergie. Und die beziffern sie in ihren gleichungen als 1. Also nach Thales A mal A plus B mal B ist C mal C (~keine Quadratfunktion). (->C = 1 ). Dies lässt sich auch in 3te 4te oder später mit ψ(x) in die Xte diemension ausweiten.

    (Innerhalb dieses Absatzes kam in mir die Frage auf: Haben sie den Artikel vieleicht verändert? Letztendlich kann man an den Kommentaren sehn das meine Vorschreiber vor knapp 3 jahren entrüstet waren und sie zwischendurch die zündende Idee hatten. Wenn ja war ihre idee gut!)

    Im folgenden möchte ich anmerken das mich ein Absatz schon verwirrt hat.

    “Bei einer Messung passiert also etwas Seltsames – der Zustand eines Systems ändert sich durch die Messung schlagartig. Das ist bereits die “spukhafte Fernwirkung”, über die sich Einstein so echauffiert hat – stellt euch vor, “hier” wäre genau hier, und “da” wäre bei Alpha Centauri. Wenn ihr das Teilchen jetzt hier entdeckt, dann ändert sich der Zustand >>(VON WAS? (Des Quants? Ist dies ein 2ter Quant?)<< bei Alpha Centauri schlagartig, obwohl das ein paar Lichtjahre sind und sich doch keine Signale mit Überlichtgeschwindigkeit ausbreiten dürfen "

    Ändert das Teilchen nun seinen Zustand nach Alpha Zentrauri und ist nun dort? Wird der Zustand Apha Zentauri verändert? ~ Allgemein finde ich es schön verbildlicht. Verstehen was genau passiert tu ich aber nicht 🙂 Was genau ist los?

    Frage 2:

    Bisher konnte unser Teilchen nur an ganz bestimmten Orten sein – hier, da oder dort. Das ist natürlich nicht besonders realistisch – ein echtes Elektron kann sich ja so ziemlich überall rumtreiben, nicht bloß an ganz bestimmten Orten. Elektron gleich Quant!?

    Ich hielt Quanten bisher nur für eine Form von Energie. Irgenteine Art von "Schwingung/Teilchen" dessen Gewicht nonexistent ist. (Photon) Ich nehme an das sie das Elektron nur Stellvertretend für einen Quanten nennen, aber da kenne ich mich nun wirklich nicht aus.
    Vieleicht wäre die "Definition" Quant mal sehr interessant. https://www.quanten.de/wassindquanten.html. Um ehrlich zu sein hat mich dieser Zusatz artikel ebenfalls verwirrt. Kleinstes Teilchen Quant. Eher eine Verwirrende Definition. Bestehen nun Atome aus Quanten… war da nicht irgentwas von Quarks und Co? (Pun intended.) bestehen nun Quarks aus Quanten oder wie ist meine Weltsicht noch weiter zu ruinieren?
    Um ehrlich Zu sein Blicke ich im Allgemeinen mit den Quanten noch nicht durch. Es gibt ja Photonen und Phononen und und und. Was genau ist nun ein Quant und wo soll ich ihn in meinem Mikrokosmos anordnen?

    Zueletzt um die Lange geschichte noch schön zu runden möchte ich einen Einleitungsgedanken wieder aufgreifen. Es ist ja bekannt das das Unternehmen Google auf den Quantencomputer hinauswill. Jedoch würde mich eine Technische umsetzung des Experimentes interessieren. Wie lese ich den Zustand eines Quants? Warum ich so genau nachfrage ist, das ich irgentwo gelesen habe, das Kohlenstoff sowie Wasserstoff unter einfluss starker magnetfelder ein relativ stabilen Quantenzustand… irgentwo ausbilden sollen, und das irgentwelche versuchsquantenrechner (20 oder weniger Qubits) auf Basis dieser Atome in einer Molkülstrucktur arbeiten. (Ist das Blödsinn?)
    Wäre ja aus der Sicht eines Chemikers durchaus interessant. 🙂

    So ich bin erstmal fertig~ Mein hirn ist gegrillt und mein Magen knurrt. Essenszeit! (Pun intended)

  132. #132 MartinB
    2. August 2016

    Sorry, hat etwas gedauert, war in Urlaub.

    @Franz
    Danke für’s Lob. Das mit denLinks vergesse ich leidermeist, aber mit der Suchfinktion sollte es schnell zu finden sein. Die Serie sind auch unter dem Link “Artikelserien” rechts aufgelistet, das nehme ich immer selbst, um alte Teile wiederzufinden.

    @Patrick
    ” da ich finde das Quanten (oder das Verständnis von ihnen) ein grundlegenderes Gerüst für unsere welt als das Borsche Atommodel sein könnten. ”
    Ja klar. Das Bohrsche Atommodell hat letztlich nur knapp 10 Jahre gehalten, dann wurde es von der “richtigen” Quantenmechanik abgelöst.

    “Was aber Zählt ist die Entfernung zum “Nullpunkt” des Systems (Koordinanten Ursprung) Die Entfernung von diesem Mittelpunkt bis zum Kreißäußeren beziffert die Quantenenergie.”
    Nein, das ist die Gesamtwahrscheinlichkeitsamplitude (die deshalb 1 sein muss). Mit der Energie hat das nichts zu tun.

    “Im folgenden möchte ich anmerken das mich ein Absatz schon verwirrt hat.”
    Das teilchen hatte vorher eine Wahrscheinlichkeit, bei Alpha Centauri zu sein. Die muss ich ja irgendwie als mathematische Funktion schreiben können – und diese Funktion ändert sich, wenn ich das teilchen hier messe, eben dort schlagartig zu Null.

    Eine ganz scharfe Definition der Begriffs “Quant” gibt es in der Physik nicht, soweit ich weiß.Da das kein Begriff ist, der durch eine Zahl beschrieben wird, braucht man auch keine ganz scharfe Definition. Tatsächlich taucht der Begriff auch in Büchern der Quantenmechanik nicht so sehr oft auf -eigentlich nur dann, wenn es um diskrete Energiemengen geht (so wie z.B., wenn ein Elektron aus einem angeregten Zustand in einer weniger angeregten übergeht und dabei ein Photon aussendet.)

    Was die Messungen angeht, das ist nicht so einfach, aber ich habe mal in einem anderen Artikel erklärt, wie man z.B. Photonen misst:
    https://scienceblogs.de/hier-wohnen-drachen/2012/10/13/nobelpreis-wie-zahlt-man-photonen-ohne-sie-zu-zerstoren/

  133. #133 Physik-Fan
    2. August 2016

    @Fitz #126, #128

    Der Hinweis von Aldemarin auf Doppelspaltexperiment trifft es genau. Wie Feininger einmal sagte (sinngemäß) steckt darin schon das ganze Geheimnis der Quantenphysik. Es geht natürlich noch raffinierter, z.B. mit dem Mach-Zehnder-Interferometer, was Zeilinger in “Einsteins Schleier” ausführlich diskutiert. Überhaupt der Name Zeilinger sollte hier fallen. Er (und seine Arbeitsgruppe) beschäftigen sich so intensiv wie kaum jemand sonst mit Grundfragen der Quantenphysik. Zeilinger hat in mehreren Sachbüchern darüber geschrieben. Man könnte ihn als den “Quanten-Guru” bezeichnen.

    Aber auch er reitet ständig die Welle-Teilchen-Paradoxien. Mich beschleicht immer wieder das Gefühl, dass das nicht das letzte Wort ist, dass es doch irgendwo eine plausiblere Interpretation gibt, dass man den richtigen Dreh noch nicht gefunden hat. Ich denke, auch anderen geht es so. Klar, Leute wie Einstein, Heisenberg, Schrödinger, Feynman, Bell haben intensiv darüber nachgedacht und kamen zu keinem befriedigenden Schluss. Also ist es letztendlich vielleicht doch nicht auflösbar.

    Betrachten wir das Doppelspaltexperiment. Warum sollte man sagen, dass das Elektron quasi durch beide Spalte gleichzeitig gegangen ist (oder im Mach-Zehnder-Interferometer beide Wege, den ohne Bombe und den mit dieser)? Man beobachtet das Elektron nicht dort. Das was man sieht ist das Auftreten eines Elektrons irgendwo am Detektorschirm und dass über längere Zeit hinweg die Elektronen sich in der Art eines Interferenzmusters ansammeln. Das ist auch dann so, wenn im Mittel nur ein Elektron in der Versuchsanordnung ist. Durch den Doppelspalt ist anscheinend eine Welle gelaufen, kein Teilchen. Nur am Detektorschirm sieht man ein Teilchen oder präziser gesagt, ein örtlich lokalisiertes, d.h. teilchenartiges Phänomen. Zum Wellenbild passt das nicht so recht, aber die “Materiewelle” wäre halt anders, sie wechselwirkt punktförmig, in einer örtlich nicht vorhersagbaren Weise. Warum sie das tut, bleibt vorerst unklar, aber ihr Verhalten ist nun mal so.

    Kurz gefasst würde das bedeuten Ausbreitung = Welle, Wechselwirkung = Teilchen, womit das Hin-und-her-Lavieren zwischen Welle und Teilchen weg wäre. Mir ist schon klar, ganz so glatt läuft die Sache nicht, es gibt Ausbreitungsphänomene, die teilchenartig interpretiert werden können. Da ist der Compton-Effekt (der ja zur Akzeptanz der Photonenhypothese Einsteins führte), aber dabei ist eine Wechselwirkung erfolgt, also nicht nur reine Ausbreitung. Ein anderes Beispiel ist die Einsteinsche Interpretation von Licht als ein Gas mit einer speziellen Statistik, woraus das Planksche Strahlungsgesetz folgt.

    Teilchenmodellierung im Wellenbild gib es aber schon. In der QFT sind Teilchen Sekundärphänomene des Quantenfeldes, werden über die Lagrangedichte des Feldes modelliert. Teilchen haben keine Existenz im Sinne einer individuellen Historie. Man kann prinzipiell nicht sagen, dass man es bei einer beobachteten Bahn mit ein und demselben Teilchen zu tun. Das wäre eigentl. schon eine Auflösung des Welle-Teilchen-Paradoxons, wenn ich es richtig verstehe. Schon bei Schrödinger tauchen Teilchen als Pakete der Wellenfunktion auf, als kompakte Überlagerung vieler Wellenfunktionen. Allerdings sind solche Pakete nicht stabil. Sie laufen auseinander, weil die einzelnen Wellenfunktionen sich mit unterschiedl. Geschwindigkeit bewegen.

  134. #134 Physik-Fan
    2. August 2016

    soll natürlich “Feynman” statt “Feininger” heißen

  135. #135 MartinB
    2. August 2016

    @Physik-Fan
    “In der QFT sind Teilchen Sekundärphänomene des Quantenfeldes, werden über die Lagrangedichte des Feldes modelliert. Teilchen haben keine Existenz im Sinne einer individuellen Historie.”
    Genau deswegen sehe ich auch eigentlich kein Problem im Welle-Teilchen-Dualismus, sondern eher im Messvorgang/ Kollaps der WF. Der ist in meinen Augen das eigentliche Problem, WTD ist nur eine halbwegs anschauliche Umschreibung bzw. ein Beispiel dafür.

  136. #136 Physik-Fan
    3. August 2016

    @MartinB
    Genau deswegen sehe ich auch eigentlich kein Problem im Welle-Teilchen-Dualismus, sondern eher im Messvorgang/ Kollaps der WF. Der ist in meinen Augen das eigentliche Problem, WTD ist nur eine halbwegs anschauliche Umschreibung bzw. ein Beispiel dafür.

    Der Kollaps der WF ist dann ein Problem, wenn man sie als eine physikalische Welle zu interpretieren versucht. In der Kopenhagener Deutung, wo der Kollaps ja herkommt, ist es aber nicht so. Da ist die WF was rein Logisches. Sie beschreibt die Ausbreitung von Wahrscheinlichkeitswerten und der Kollaps ist dann nur die Feststellung, dass man mit der Messung das Teilchen an einem Ort feststellt, dass es dort ist (Wahrscheinlichkeit = 1) und nirgendwo anders (Wahrscheinlichkeit = 0). Für die “Wahrscheinlichkeitsfunktion” (so nennt sie Heisenberg) ist die abrupte, überall zugleiche Zustandsänderung kein Problem, für sie gibt es keine physikalischen Restriktionen dafür.

    An dieser Interpretation der WF entzündet sich die Kritik an der Kopenhagener Deutung. Eine Funktion mit logischen Werten, macht das Sinn in der Physik? Zumal sich über das Betragsquadrat ja messbare Werte ergeben, die Funktion aber z.T. ein physikalisch fragwürdiges Verhalten hat. Eigentl. ist so die WF keine tiefere physikalische Struktur, sondern rein phänomenologisch orientiert, etwas, das die Beobachtungen wiedergibt. Von den Gründervätern der Kopenhagener Deutung war das genau so beabsichtigt. Sie meinten, man kann nur den Grad unserer Kenntnis beschreiben, aber nicht irgendwelche zugrundeliegenden physikalischen Elemente. Die Ausbreitung der WF kann nicht mit dem Mitteln der klassischen Physik, mit unser Anschauung beschrieben werden (s. “Quantentheorie und Philosophie”, Heisenberg).

  137. #137 MartinB
    3. August 2016

    @Physik-Fan
    Klar, kann man so sehen, aber dann ist die Qm nicht-lokal und nicht-realistisch, d.h. wir verwenden zum beschreiben der Natur Entitäten, von denen wir annehmen, dass ihnen in der Natur nichts entspricht.
    Finde ich auch sehr problematisch.

  138. […] Bei meinen Beträgen beispielsweise zur Quantenmechanik gibt es sehr einfache Darstellungen, aber auch solche, die schon dicht an dem dran sind, was man an der Uni lehrt. Bei Diskussionen über […]

  139. #139 Eugen
    6. April 2017

    Eine kurze Erwähnung, dass die Zustände einen Hilbertraum bilden wäre evntl. sinnvoll.

  140. #140 MartinB
    7. April 2017

    @Eugen
    Warum? Diejenigen, die wissen, was ein Hilbertraum ist, wissen vermutlich auch, dass die Zustände einen bilden, diejenigen, die es nicht wissen, sind hinterher auch nicht schlauer.

  141. […] Im ersten Teil des Comics geht es um eine Einführung in die Quantenphysik auf Grundlage dieses hervorragenden Artikels (Quantenmechanik verstehen? via ScienceBlogs) von Martin Bäker. Weitere Quellen und Lesetips […]

  142. #142 mischa
    egal
    25. März 2018

    Ich kannn dir absolut folgen, leider kann ich das nahe zu immer. Es regt mich auf, dass meine hochbegabung im logischen und logisch-abstraktem Denken nicht gefordert wird. Ich will jetzt nicht rum heulen aber mein bewisenes Genie wurde noch nie an seine Grenzengebracht. Und das ich nur auf grund meines Sozialenufeldes und meines Rufes(ansehen) nicht die chance bekomme die ich verduent habe, macht mich fertig. Ich bin einfach zu “anders” um mich einer normalen Lehre wie es in der Schweiz ùblich ist hin zu geben. Vileicht klingt es eitel aber ich bin der Überzeugung, dass ich grosses bewirken könnte, da ich ein mensch bin der unter “asbergersyndrom”(mag nicht googeln wie man das schreibt) agiert. Ich würde viele neue möflichkeiten auf die Sicht der Dinge(und auch der quantenmechanik) hervor bringen. Leider lässt unser Bildungssystem nicht zu “abzukürzen”, was fùr einen Menschen wie mich sehr frustierend ist, da ich mich fùhle als mùsste ich meine inteligentz (auf dem weg zu meinem Ziel,)immer und immer wieder unter beweis stellen. So das am ende nicht genug (oder viel weniger) Zeit bleibt, dass mir jemand glaubenschenkt und wir projekte “des weltraums-bevôlkern” zu spät in an spruch nehmen und wir die “wahreheit(die immerhin alle zufriedenstellt)” erleben könnten. An all fie Autodidakten da drusen:”ich glaube es liegt an uns die menschheit voran zu treiben, auch wenn es unheimlich viel aufwand kostet, ist es das wert, wenn wir(die menschheit)” eine antwort hat.

  143. #143 mischa
    25. März 2018

    bzw ich habe etwasgetrunken, entschuldigt bitte meine rechtschreib weise

  144. #144 MartinB
    25. März 2018

    @mischa
    Falls du – jetzt vielleicht wieder nüchtern – das ernst meint: Heutzutage im Internet-Zeitalter kannst du dich in Foren (wie auch hier oder bei physicsforum usw) problemlos an Diskussionen beteiligen und dein Können zumindest dort unter Beweis stellen. Dann kannst du zumindest erst mal testen, was du wirklich kannst.

  145. #145 Peter
    4. Juli 2018

    Toller Artikel! Hat Spaß gemacht ihn zu lesen und die Beispiele waren sehr anschaulich

  146. #146 MartinB
    4. Juli 2018

    @Peter
    Fein, freut mich zu hören.

  147. #147 Con
    21. August 2018

    Hallo,

    ich war auf der Suche nach einer Antwort auf die Frage, was die Quantenmechanik ist, welches Gebiet sie beschreibt und was man damit machen kann. Jetzt habe ich diesen Artikel gelesen und auch den Artikel “Quantenmechanik – die beliebtesten Phrasen”.

    Möglicherweise liegt es an mir, ich bin Laie, aber aus meiner Sicht setzen beide Aritkel bereits voraus, dass man weiss, was die Quantenmechanik überhaupt ist und auch, dass man überzeugt ist, dass sie überhaupt ein tatsächlich bewiesenes Gebiet ist.

    Dieser Artikel hier beginnt bereits damit, dass ein Teilchen mehrere Zustände haben kann (Es ist hier und da gleichzeitig). Der Artikel erklärt aber nicht, was die Quantenmechanik ist und ob sie stimmt.

    Beispiel Mathematik. Jemand fragt, was Mathematik ist. Dann würde ich diesem jemand als erstes erklären, das man damit Mengen berechnen kann. Wenn er einen Haufen Äpfel hat zum Beispiel, dann kann man mit Mathematik diese auf Personen aufteilen, die Anzahl berechnen oder mehrere Haufen bilden. Im zweiten Schritt würde ich ihm beweisen, dass die Mathematik keine Fantasie ist, sondern Regeln hat, die er immer anwenden kann. Das 2 + 2 = 4 ist, egal ob er Äpfel hat oder sein Lehrer ein Tyrann ist. Und erst im dritten Schritt würde ich ihm erklären, wo der Unterschied zwischen Addition und Subtraktion ist.

    Dieser Artikel hier macht aber nur den dritten Schritt, nach dem Motto: “Das hier sind die Regeln der Addition und Subtraktion, was Mathematik ist und das diese bewiesen ist, setze ich als Wissen voraus.”
    Wofür man diese Regeln benutzt und ob sie überhaupt stimmen, wird nicht erklärt.

    Wie gesagt, ich bin Laie, ich habe nach einer Erklärung gesucht, was Quantenmechanik überhaupt ist. Die Biologie ist die Lehre des Lebens. Die Genetik ist die Lehre von der Vererbung. Die Evolution die Lehre der Entwicklung des Lebens. Genetik und Evolution sind also Teilbereiche der Biologie.

    Die Physik ist die Lehre von den Naturphänomenen. Die Mechanik ist die Lehre von der Bewegung von Körpern. Und ein Quant soll ein Objekt sein, dass durch den Wechsel eines Zustandes erzeugt wird.

    Was ist jetzt die Quantenmechanik? Was macht man damit? Und wer hat bereits bewiesen, dass ein Teilchen sowohl hier als auch da sein kann?

    Mein Wissensstand (wenn man das so nennen darf) über Quantenmechaniker ist nämlich folgender: Das sind irgendwelche Irren, die Teilchen und einem nicht näher definierten “Universum” ein Bewusstsein zusprechen, welches sich zwischen Zuständen aktiv entscheiden kann. Deshalb kann in der Überzeugung der Quantenmechaniker eine Katze auch sowohl leben als auch gleichzeitig tot sein.

    Freue mich über Links zu Artikeln, welche die Quantenmechanik für Laien verständlich erklären. Danke sehr!

  148. #148 MartinB
    21. August 2018

    @Con
    Klick mal rechts bei den Artikelserien, da gibt es noch mehr zur QM. Die QM ist genau das, was ich auch in diesen Artikeln beschreibe: Eine Theorie, die das Verhalten von Teilchen wie Elektronen korrekt beschreibt. Und für ne grundlegende Definition kannst du auch bei Wikipedia nachgucken.
    Die Serie zum Beispiel zur Schrödingergleichung erklärt auch Beispiele, ebenso die weiteren Teile dieser Serie.
    Das was du im vorletzten Absatz schreibst, ist nicht mal mehr falsch…

  149. #149 Con
    21. August 2018

    @MartinB, 148

    Da gibt man sich Mühe, gibt ein umfassendes Beispiel, ein paar Vergleiche und was kriegt man zurück? “Klicke mal die Artikel durch und frag Wikipedia”.
    Wieso haben Sie diesen Dreiteiler hier eigentlich geschrieben und versucht in den “quantenmechanischen Zustand einzuführen”? Sie hätten doch auch schreiben können: “Wenn ihr immer noch nicht kapiert hat, worum es geht, lest Wikipedia und meine anderen Artikel. Wer es dann noch nicht versteht, ist selber schuld.”

    Sie machen sich hier (link) über Leute lustig, die nicht verstehen, was es mit der Quantenmechanik auf sich hat. https://scienceblogs.de/hier-wohnen-drachen/2012/02/05/ich-bin-jetzt-quantenheilungsskeptiker/

    Dass jeder Laie zwangsläufig bei solchen Quacksalbern landet, wenn Wissenschaftler wie Sie keine Lust haben, etwas laienhaft zu erklären (oder wenigstens zu empfehlen, an welcher Stelle des Internets man auf seriöse leicht verständliche Informationen stößt), ist Ihnen doch hoffentlich klar, oder?

    Und jetzt schreiben Sie auch noch, dass es tatsächlich möglich ist, in der realen Welt(!) gleichzeitig lebendig und tot zu sein.

    Sagen Sie mir, für mich als Laie, woran kann ich erkennen, dass Ihre Texte und Aussagen seriöser sind, als die von den Quacksalbern? Oder anders gesagt, wenn Sie mal für einen Moment so tun, als hätten sie nicht in Elementarprozessen promoviert, wieso sollten Sie als Laie “Bündel von Schwingungen” für unseriöser halten als “Viele-Welten-Theorie”?

    Ich mag vielleicht wie ein Troll klingen oder unhöflich, aber ich hatte mir Mühe gegeben, freundlich zu erklären, dass ich Laie bin. Dass ist keiner der promovierten Physiker bin, die hier mit Ihnen auf einer Welle diskutieren können. Und zurück bekam ich eine abweisende Antwort aus dem metaphorischen Elfenbeinturm.
    Um bei meinem Beispiel zu bleiben, der Laie fragt “Was ist Mathematik?” und Sie antworten: “Lies erstmal den Artikel über Mathematik bei Wikipedia, hier im Blog sind ein paar Beispiele zu Differentialgleichungen und Mengenlehre, die helfen dir sicher weiter. Und dass du einen Apfel haben kannst, auch wenn du keinen hast, ist nicht mal falsch…”

    Gute Nacht.

  150. #150 MartinB
    21. August 2018

    @Con
    Zunächst mal:
    1. Es ist ein bisschen so, als würdest du zu einem Mercedes-Händler gehen und fragen, warum es dort keine preisgünstigeren Autos gibt. Dieser Artikel hat eine Zielgruppe: Leute, die schon versucht haben, die QM zu verstehen. Das wird ja in der Einführung ziemlich deutlich gemacht, darum geht es. Wer sich noch nie damit beschäftigt hat, ist vielleicht mit anderen Texten besser bedient.
    2. Ich schreibe diese Artikel in meiner Freizeit. Ich tue das, weil ich es wichtig finde, anderen zu erklären, wie die Wissenschaft funktioniert, aber ich bin da in keiner Bringschuld.

    “Und jetzt schreiben Sie auch noch, dass es tatsächlich möglich ist, in der realen Welt(!) gleichzeitig lebendig und tot zu sein.”
    Wenn du mir sagst, wo ich das geschrieben habe, dann erkläre ich, wie ich das gemeint habe. (Ich bin mir aber ziemlich sicher, dass ich das nirgends geschrieben habe…)

    “Ich mag vielleicht wie ein Troll klingen oder unhöflich, aber ich hatte mir Mühe gegeben, freundlich zu erklären, dass ich Laie bin”
    Richtig. Und ich habe dir deine Frage “Was ist QM” so kurz, knapp und laienhaft wie möglich beantwortet:
    “Eine Theorie, die das Verhalten von Teilchen wie Elektronen korrekt beschreibt. ”
    Das ist tatsächlich auf den Punkt gebracht eine gute Antwort (man kann sich über Details lange streiten, aber als erste Erklärung ist es auf jeden Fall brauchbar.)
    Weiter habe ich das “freue mich über links” ernstgenommen und neben meinen Artikelserien auf Wikipedia verwiesen – so schlecht ist die Seite nämlich nicht, und der erste Absatz gibt eine sehr gute Definition davon, was die Qm ist und wozu sie gut ist, in meinen Augen genau das, was du wolltest.

    Wenn du konkreter nachfragst und sagst, was dir unklar ist, bekommst du vermutlich auch ne konkretere Antwort. Bisher weiß ich nur, dass du eine Definition suchst, was QM überhaupt ist, die habe ich dir (kurz, da ich auch nicht mal eben schnel nen 2000-Wort-Artikel aus dem Ärmel schütteln kann) gegeben bzw. verlinkt.

    Im übrigen: Wenn du schon weißt, dass du unhöflich klingst, wie wäre es dann, einfach den Ton zu wechseln oder dich zumindest dann nicht zu echauffieren, dass mein Ton ebenfalls nicht gerade der allercharmanteste ist? Wie man in den Wald hineinruft…

  151. #151 Bullet
    24. August 2018

    Der Artikel erklärt aber nicht, was die Quantenmechanik ist und ob sie stimmt.

    Lol? Was war’n das?

  152. #152 MartinB
    25. August 2018

    @Bullet
    Huhu [hektisch-wink], lange nicht gesehen…

    Ansonsten: Keine Ahnung. Vielleicht irgendwer, der vor sich selbst die vorgefasste Meinung rechtfertigen möchte, dass Wissenschaftlerinnen alles arrogante Elfenbeinturmbewohnerinnen sind,

  153. #153 Stefan K
    25. Oktober 2018

    Hallo. Ich kenne dieses Blog nun schon länger und lese fasziniert mit, aktuell versuche mich gerade durch die ganzen alten Artikelserien zu lesen, da mich die Themen und Physik allgemein sehr faszinieren. (Leider habe ich vor langer Zeit das falsche studiert, also reichts nur fürs Niveau interessierter Laie). Ich hoffe Feedback bzw. Fragen sind auch nach Jahren noch erwünscht bzw erlaubt. was ich nicht ganz verstehe: wieso ist es das Quadrat der einzelnen Wahrscheinlichkeiten, die 1 ergeben müssen und nicht die Wahrscheinlichkeiten selbst? (Oder anders ausgedrückt, wieso liegen die zustände am einheitskreis und nicht auf der Gerade zwischen a und b?) Habe schon versucht es selbst rauszufinden, bin dabei auf Wahrscheinlichkeitsamplituden gestoßen, aber letztendlich stecke ich in einer Sackgasse, weil ich mir darunter nicht wirklich etwas vorstellen kann bzw. nicht weiß, was die Wahrscheinlichkeitsamplitude ist und was sie von “der Wahrscheinlichkeit” unterscheidet.

  154. #154 MartinB
    25. Oktober 2018

    @StefanL
    “wieso ist es das Quadrat der einzelnen Wahrscheinlichkeiten, die 1 ergeben müssen und nicht die Wahrscheinlichkeiten selbst? ”
    Dafür gibt es keine Erklärung – es ist das, was die Natur korrekt beschreibt. Zustände werden durch die Amplituden beschrieben, für die Wahrscheinlichkeiten muss man die quadrieren. Man kann das nicht aus irgendwelchen fundamentaleren Theorien ableiten oder so. Niemand kann sich unter eine Wahrscheinlichkeitsamplitude was vorstellen…

  155. #155 Stefan K
    25. Oktober 2018

    Danke für die rasche Antwort, die mich inhaltlich im Übrigen auf sehr egoistische Art freut, immerhin hab ich gedacht, ich hab irgendwas banales nicht verstanden oder übersehen, aber “niemand kann sich unter einer Wahrscheinlichkeitsamplitude was vorstellen” find ich sehr viel besser als alle können das, nur ich nicht 😉

  156. #156 MartinB
    25. Oktober 2018

    @StefanK
    Irgendwo ist mit dem Verstehen halt schluss und man muss die Dinge akzeptieren. Oder wie ein Physikprof es mal sagte:
    “Das können Sie nicht verstehen, daran können Sie sich nur gewöhnen.”

  157. #157 Alderamin
    25. Oktober 2018

    @MartinB

    Dafür gibt es keine Erklärung – es ist das, was die Natur korrekt beschreibt.

    Hmm, ein bisschen dünn… zuerst hätte ich gesagt, Amplituden können ja auch negativ sein, Wahrscheinlichkeiten nicht, und dann sagt Wikipedia, sie können auch komplex sein, das schreit nach Quadrieren.

    Nach etwas Googelei fand sich dann diese nette Seite von Sean Carroll, der das Quadrat mit Everett begründen kann. Was hälst Du davon?

    Everetts “viele-Welten-Theorie” ist ja, soviel ich weiß, gleichwertig mit anderen quantenphysikalischen Deutungen, d.h. wenn etwas da gilt, muss es auch in den anderen Deutungen richtig sein. Dann wäre obiges eine Erklärung auch für Kopenhagen.

    Wäre vielleicht einen Blogartikel wert 😉

  158. #158 MartinB
    25. Oktober 2018

    @Alderamin
    Auf die Schnelle muss ich schlicht zugeben, dass ich es nicht verstehe. Klar, Wahrscheinlichkeiten müssen positiv und normiert sein, und wenn man mit Zuständen in Hilberträumen anfängt, ist es vermutlich nicht leicht, einen anderen Weg zu finden, immer konsistente wahrscheinlichkeiten zu bekommen – aber dann kann ich natürlich fragen “Warum Hilberträume”?
    Und Dinge wie
    “ESP simply asserts that our knowledge is separable:”
    scheinen mir mehr oder weniger vorauszusetzen, was wir zeigen wollen – denn prinzipiell spricht nichts dagegen, dass ich dank Quantenverschränkung bei Alpha Centauri das beeinflusse, was hier passiert, oder nicht?
    Vielleicht bin ich aber im Moment auch nur nicht tief genug drin, um zu sehen, was da für eine Idee drinsteckt, und verstehe was falsch…

  159. #159 alex
    25. Oktober 2018

    @Alderamin:
    Soweit ich weiß geht in derartige Argumentationen immer ein, dass der Zustandsraum ein Hilbertraum ist. Das ist mathematisch natürlich sehr schön, weil Hilberträume eine reichhaltige Struktur haben. Aber ich sehe physikalisch keinen Grund, weshalb das so sein sollte. (Außer dass die Ergebnisse die man so erhält extrem gut zu den Experimenten passen.)

    Und wenn man erst einmal einen Hilbertraum hat, dann ist das Betragsquadrat ziemlich natürlich. Es kommt ja z.B. in der Norm oder bei orthogonalen Projektionen vor.

    Wenn hingegen in den Zuständen statt Wahrscheinlichkeitsamplituden direkt die Wahrscheinlichkeiten ständen, dann hätte man keinen Hilbertraum. Sondern den Raum der Wahrscheinlichkeitsmaße auf einer gewissen Grundmenge an “Basis-Zuständen”. Und Superpositionen wären nicht Linearkombinationen von Zuständen sondern Konvexkombinationen. (Man könnte natürlich einen solchen Zustandsraum auf den von den Basiszuständen aufgespannten linearen Raum ausweiten. Aber soweit ich sehe wäre das dann kein Hilbertraum, weil es kein natürliches Skalarprodukt gäbe.)

  160. #160 Alderamin
    25. Oktober 2018

    @MartinB

    Ich hab’ das Carroll-Paper mal kurz aufgemacht, das ist lang, sieht aber verhältnismäßig harmlos aus. Müsste man wohl mal lesen.

    Was ich aus dem Blogartikel von Carroll verstanden habe ist, dass sein Argument so geht: bei den Viele-Welten ist jeder Ablauf grundsätzlich deterministisch. Die Wahrscheinlichkeit der Wellenfunktion ergibt sich daraus, wie wahrscheinlich ich mich mit einem bestimmten Beobachter, der ein bestimmtes Ereignis beobacheten wird, identifiziere, wenn ich sozusagen aus der Menge aller Beobachter im gleichen Anfangszustand einen zufällig ziehe. Wie wahrscheinlich ist es, dass ich einer der Beobachter bin, die den Spin des Elektrons nach oben sehen? Oder das Einschlagen eines Photons nach Passieren eines Spalts am Ort x?

    Man könnte naiverweise annehmen, dass es für jeden möglichen Ablauf genau einen Beobachter gibt, dann wären alle Folgezustände gleich wahrscheinlich und man bräuchte nur die Wege zu gleichen Zuständen abzuzählen, um ihre Wahrscheinlichkeiten zu ermitteln.

    Davon gibt es aber beliebig viele, weil es welche gibt, wo in China ein Sack Reis umfällt, und solche, wo das nicht der Fall ist. Daher nimmt Carroll an, dass Wege, bei denen anderswo unkorrellierte Dinge passieren, nicht betrachtet zu werden brauchen, was ein vernünftig erscheinendes Axiom ist.

    Und dann folgt aus einer Herleitung, die im Paper aber nicht im Blog ausgeführt ist, dass nur das Quadrieren der Amplituden zu einer derartigen Gewichtung der Wahrscheinlichkeiten führen kann, die unkorrelierte Wege beliebig vernachlässigbar macht.

    Dazu müsste man dann das Paper lesen…

  161. #161 Alderamin
    25. Oktober 2018

    @Alex

    Da muss ich passen und mich darauf verlassen, dass Carroll weiß, was er da rechnet. Ich hab’ das hier nur mal in die Runde geworfen, weil ich dachte, vielleicht steht da ja, wie vom Autor behauptet, eine nachvollziehbare Erklärung drin, warum Borns Gesetz gelten muss, jedenfalls in der Everett-Interpretation. Was er da im Paper rechnet, habe ich, wie gesagt nur mal kurz angeschaut, die Formeln sehen einfach aus, aber das muss nichts heißen…

  162. #162 Niels
    25. Oktober 2018

    @MartinB @alex

    wenn man mit Zuständen in Hilberträumen anfängt, ist es vermutlich nicht leicht, einen anderen Weg zu finden, immer konsistente wahrscheinlichkeiten zu bekommen

    Und wenn man erst einmal einen Hilbertraum hat, dann ist das Betragsquadrat ziemlich natürlich.

    Man kann sogar mathematisch zeigen, dass das gar nicht anders geht.
    Nennt sich Gleason’s Theorem.
    https://en.wikipedia.org/wiki/Gleason%27s_theorem

    @Alderamin

    Everetts “viele-Welten-Theorie” ist ja, soviel ich weiß, gleichwertig mit anderen quantenphysikalischen Deutungen, d.h. wenn etwas da gilt, muss es auch in den anderen Deutungen richtig sein. Dann wäre obiges eine Erklärung auch für Kopenhagen.

    Nö, das ist wirklich nur eine Erklärung für Viele Welten.

    Für etwas anderes als philosophische Sophisterei ist das alles außerdem völlig irrelevant.

    Wenn diese Idee richtig wäre, könnten Viele Welten Anhänger ab jetzt behaupten, dass sie die einzige Interpretation vertreten, die die bornsche Regel aus einem tieferliegenderen Prinzip ableitet.
    (Wobei dann natürlich grundsätzlich statt der bornschen Regel eben dieses Tieferliegendere postuliert werden muss, nämlich speziell hier das “Epistemic Separability Principle”.)

    Dadurch kann man aber natürlich immer noch nicht messen, welche Interpretation denn jetzt die Richtige ist.

    Das einzige, dass sich ändert, ist, dass man als Viele Welten Anhänger nun argumentieren kann, dass aus philosophischer Sicht jetzt noch mehr für Viele Welten spricht als vorher.

    Und die Anhänger anderer Interpretationen werden das daraufhin dann einfach weiter völlig anders sehen…

  163. #163 MartinB
    25. Oktober 2018

    @Niels
    Danke, dann bin ich beruhigt und bleibe bei meiner Interpretation der QM, die besagt, dass wir schlicht keine Ahnung haben…

  164. #164 Niels
    27. Oktober 2018

    Hm, ich wollte die Diskussion eigentlich nicht abwürgen.

    Ich persönlich finde solche Dinge ja durchaus sehr interessant.
    Gerade deswegen halte ich es aber für ziemlich wichtig, dass man absolut klar stellt, dass man sich gerade eigentlich mit Philosophie beschäftigt und nur im weitesten Sinne noch Physik betreibt.

    Ohne Kontext kommen Floskeln wie “philosophische Sophisterei” aber wahrscheinlich deutlich negativer rüber als eigentlich gemeint…

  165. #165 MartinB
    27. Oktober 2018

    @Niels
    Ich denke nicht, dass du da was abgewürgt hast.
    Interessant finde ich sowas prinzipiell auch, aber ich habe immer den Eidnruck, dass in diesem Bereich oft viel hype um wenig Erkenntnis gemacht wird – da löst angeblich Dekohärenz plötzlich alle Probleme, oder viele Welten usw.

  166. #166 Björn
    Wien
    2. November 2018

    Guten Tag,
    Soeben bin ich auf Ihren sehr gut ausgearbeiteten Artikel gestoßen. Danke dafür.
    Wenn ich das richtig verstanden habe, sind vor der Messung des Zustandes des Teilchens prinzipiell alle Zustände möglich die das Teilchen annehmen kann und die Messung sorgt dafür dass das Teilchen diesen spezifischen Zustand annimmt. Es wird zu einem entweder oder. Jetzt meine Frage. Könnte es nicht auch sowohl als auch sein nur einfach in so einer Geschwindigkeit die nicht messbar ist. Angenommen Teilchen a und b enthalten eigentlich diesselbe Information die beliebig zwischen ihnen auswechselbar ist und somit sind sie sowohl a als auch b. Ich stelle mir das so vor, dass zum Beispiel eine zeitliche Abfolge einer Bewegung die Zustände in denen die Teilchen waren und sein werden schon mit einer gewissen Wahrscheinlichkeit bestimmen und diese Information von Teilchen zu teilchen wandert also in sich keinen festen Kern hat, dabei aber stets 1 ergibt, also in diesem System möglich ist. Beste Grüße und Danke noch einmal für den tollen Artikel

  167. #167 MartinB
    2. November 2018

    @Björn
    So ganz habe ich nicht verstanden, wie du das meinst, aber eine zeitliche Abfolge gibt es da im allgemeinen nicht, dann würde man ja z.B. periodische Effekte sehen können.
    Die nächsten beiden Artikel der Serie erklären noch detaillierter, wie es funktioniert.

  168. #168 roel
    2. November 2018

    @Björn MartinB schreibt es zwar schon:

    “Die nächsten beiden Artikel der Serie erklären noch detaillierter, wie es funktioniert.”

    Das sind die beiden Artikel:

    https://scienceblogs.de/hier-wohnen-drachen/2013/01/10/quantenmechanik-verstehen-ii-noch-mehr-zustande/

    https://scienceblogs.de/hier-wohnen-drachen/2013/01/27/quantenmechanik-verstehen-iii-verschrankung/

    Und hier geht es zu Überblick über die Artikelserien: https://scienceblogs.de/hier-wohnen-drachen/artikelserien/

  169. #169 Björn
    Wien
    2. November 2018

    Super Danke vielmals. Ich werde mich da noch einmal genau einlesen und kann dann vielleicht eine präzisere Frage stellen.

  170. […] in der Quantenmechanik. Ausführlich habe ich die in dieser dreiteiligen Serie erklärt (Teil 1, Teil 2, Teil 3), deshalb gibt es heute nur eine Kurzfassung mit der […]

  171. #171 Jürgen U
    Münster
    5. Januar 2019

    Ich möchte hier mal ein paar kritische Anmerkungen zu Quantentheorie einfließen lassen.
    Ich behaupte, dass Heisenberg und seine Relationen philosophisch völlig überfrachtet worden sind. Das hat nicht unwesentlich zu der Quantenkonfusion geführt.
    Begründung: Heisenberg war in seinen jungen Jahren vom Ehrgeiz zerfressen. Dass Schrödinger die Äquivalenz seiner Wellenmechanik mit Heisenbergs Quantenmechanik bewies, machte ihn schon sauer; dass aber auch sein väterlicher Freund Max Born die wellenmechanischen Differentialgleichungen (Dgl) seinen schwerfälligen quantenmechanischen Matrizen vorzog (was alle Physiker taten), machte ihn vollends stinkig: er sprach 3 Monate nicht mehr mit Born! Nun hatte er – zum Glück – seine Relationen eingeführt und um seine Bedeutung in der Physikgeschichte – an der ihm so viel lag – nachhaltig zu etablieren, wurden diese Relationen philosophisch dermaßen hochgepuscht, dass nachher keiner mehr durchblickte – und das bis zum heutigen Tag!
    Wie so oft in der Naturphilosophie wurde auch hier Mathematik mit Ontologie (Wirklichkeit) verwechselt. Die Bedeutung der Relationen liegt eher in der Metrologie (Messlehre) als in der Philosophie. Sie beschreiben lediglich, was man im Quantenbereich erwarten kann, wenn eine nicht störungsfreie Messung durchgeführt wird – nicht mehr und nicht weniger! Heisenbergs Unschärfen beruhen auf der Wechselwirkung zwischen Objekt und Messapparatur. Dass die Objekte nur mit Unschärfe gemessen werden können, bedeutet eben NICHT, dass sie existentiell unscharf sind (wenn Quanten existentiell unscharf werden, dann deshalb, weil auf dieser Sub-Ebene ponderable diskontinuierliche Massen mit dem eigenen kontinuierlichen EM-Feld verschmelzen. Das ergibt sich aus Einsteins Masse-Energie-Äquivalenz, E=mc2; Energie/Strahlung und Massen sind zwei Erscheinungsweisen der Materie; je weiter wir in den Mikrokosmos vorstoßen, desto mehr verschwindet der makroskopische Unterschied und schließlich haben „Quanten“ eben beide Eigenschaften, je nach Kontext.) Man kann die Messungen so einrichten, dass entweder der Ort zu annähernd 100%, oder der Impuls zu annähernd 100% gemessen werden. Das Ganze ist in der Tat nur ein Messproblem und kein philosophisch-ontologisches und hat mit der Realität eben nur im engen Ausschnitt der Messapparatur bzw. der Messsituation zu tun.
    Im Übrigen war die „Entdeckung“ Heisenbergs weder neu noch fundamental, denn eine einfache logische Analyse führt dazu, dass ein Objekt nicht zu ein & demselben Zeitpunkt einen definierten = festen Ort UND zugleich einen Impuls haben kann:
    definierter fester Ort bedeutet, dass das Objekt zu diesem Zeitpunkt keine Geschwindigkeit/Impuls haben kann; Voraussetzung für einen definierten Ort ist also v=0 => p=0;
    definierter Impuls bedeutet definierte Geschwindigkeit, diese ist definiert als Ortsveränderung über die Zeit.
    Man muss also nicht Heisenberg gelesen haben, um zu erkennen, dass def. Ort & def. Geschwindigkeit komplementäre Größen sind.
    Warum scheinen diese Größen so wichtig zu sein?
    Um die Bahn eines Objektes berechnen zu können, benutzen wir Bewegungsgleichungen, z.B. x = x0 + vt; den Bahnpunkt x wollen wir berechnen, die Zeit t wird vorgegeben, der Anfangsbahnpunkt x0 und die Anfangsgeschwindigkeit v müssen zum Zeitpunkt ti bestimmt werden; m.a.W. die Ermittlung von Ort & Geschwindigkeit zum einem Zeitpunkt ist eine ganz banale Forderung der Mathematik – nicht mehr und nicht weniger. Es steckt kein Fünkchen Philosophie darin und sagt über die wirklichen Eigenschaften von Objekten kaum was aus. Diese Banalität wurde aber von den Kopenhagenern bis zur Absurdität hochgepuscht: Indeterminismus, Versagen des Kausalgesetzes, Verrücktheit dieser Theorie
    Der Höhepunkt dieses (meist amerikanischen) Quantenschwachsinns: wenn ich nicht hinschaue, gibt es auch kein Objekt! Zu sehen in vielen US-Büchern & Dokumentationen. Einstein war davon überzeugt, dass der Mond auch da ist, wenn er nicht hinschaut! Das Auge ist in der Außenwahrnehmung absolut passiv, hat absolut keine Wechselwirkung mit Objekten, kann diese infolgedessen auch nicht in die Existenz bringen!
    Ähnliches mit der Schrödinger-Gleichung: kein Mensch weiß, was da überhaupt schwingt, selbst Schrödinger nicht! Also wurde quadriert und probabilisiert und siehe da: es passt zu den Messergebnissen. Das ist eine Verzweiflungstat und keine Systematik! Und wiedermal wurde Mathematik mit Realität verwechselt und der mathematische Wahrscheinlichkeitscharakter & Indeterminismus als real existierende Eigenschaften von Objekten interpretiert.
    Wenn ich von Münster nach Malta fliege, dann gibt es eine bestimmte Unbestimmtheit, eine Wahrscheinlichkeit (und keine Sicherheit) dafür, rechtzeitig anzukommen. Diese Unbestimmtheit schwebt über mir während des gesamten Fluges. Wenn ich in Luca/Malta lande, kollabiert die Wahrscheinlichkeit zur Sicherheit! Der Kollaps der Wellenfunktion ist banale Mathematik und kein philosophisches Konzept!!!
    Wir ENTDECKEN nicht die Mathematik – wir ENTWICKELN sie; und nicht alles, was mathematisch ist, ist auch real!

  172. #172 MartinB
    5. Januar 2019

    @Jürgen
    Wenn man schon Heisenberg kritisiert, dann sollte man wenigstens keinen Kram wie
    ” Heisenbergs Unschärfen beruhen auf der Wechselwirkung zwischen Objekt und Messapparatur. ”
    schreiben –

    oder gar Unsinn wie
    “definierter fester Ort bedeutet, dass das Objekt zu diesem Zeitpunkt keine Geschwindigkeit/Impuls haben kann”
    denn das geht natürlich problemlos. Wenn ich eine Funktion x(t) habe, dann liegt der Wert von x zur Zeit t vollkommen genau fest, ohne Unschärfe.

    Nicht mal Ableitungen verstehen, aber heisenberg kritisieren…

    Weiteren ahnungslosen Unsinn von dir werde ich löschen.

  173. #173 Jürgen U
    Münster
    5. Januar 2019

    Hallo Martin,
    wenn du es dir nochmal durchliest, wirst du bemerken, dass es nicht um die Physik oder Mathematik
    der QT geht, sondern um ihre philosophische Interpretation! Die Formeln der QT sind völlig korrekt,
    sie ist eine der am meisten verifizierten Theorien.
    Aber die Kopenhagener Philosophische Interpretation ist brüchig & löchrig; es wimmelt von Postulaten,
    deren einzige Rechtfertigung darin besteht, die Messergebnisse zu bestätigen!
    Es gibt einige QT-en, deren Interpretationen wesentlich mehr Hand & Fuß haben, z.B.
    BOHM’s Deterministische QT, Ulrich HOYER’s statistisch fundierte synthetische QT oder SALLHOFER’s Arbeiten.
    Mal über den Tellerrand schauen und nicht alles nachfaseln wäre nicht schlecht.
    Wenn kritische Anmerkungen nicht erlaubt sind, bin ich hier falsch!

  174. #174 MartinB
    5. Januar 2019

    @Jürgen
    Dass es unterschiedlcihe Interpretationen gibt, ist eine Sache – ich bin auch kein Kopenhagen-Fan.
    Macht aber den Unsinn, den du oben geschribeen hast, nicht sinnvoller – wer nicht mal versteht, dass die Tatsache, dass die Ableitung einer Funktion ungleich Null ist, nicht bedeutet, dass man den Wert der Funktion an einer Stelle nicht genau kennen kann, sollte sich mit arroganten und herablassenden Bemerkungen über Leute wie heisenberg (dessen Matrizenmechanik natürlich heutzutage permanent verwendet wird, gerade in der QFT) dann doch etwas zurückhalten.
    Kritisieren ist nicht dasselbe wie Anfeinden.

  175. #175 Toni
    5. Januar 2019

    @Jürgen U:

    Bomsche Interpretation der Quantenmechanik beruht auf die Existenz von Quantenpotentiale (Quantenhypothese), die nach dem was ich weiß experimentell NICHT bestätigt wurde. Die hergeleiteten Gleichungen der Bohmsche Mechanik sind basiert auf Schrödingergleichung. Sie nimmt viel mehr Annahmen als die Quantenmechanik, und ist weitaus schwieriger vereinbar mit der speziellen Relativitätstheorie. Dass in der bohmsche Mechanik, Teilchen auf Wellen reiten, ist auch gerade nicht sehr glaubhaft oder? Es gibt wirklich gute Gründe, warum 99% der Physiker diese Theorie ablehnen.

    Das Messproblem, Quantenverschränkung, etc… werden zumindest einigermaßen durch Quantenfeldtheorie beseitigt, wenn man von Felder statt von Teilchen ausgeht. Dazu kann man einige Arbeiten im Netz finden. Auch Julian Schwinger geht in seiner QFT darauf ein.

  176. #176 MartinB
    5. Januar 2019

    @Toni
    Was die Bohm-Probleme angeht, stimme ich dir im wesentlichen zu. Die QFT sagt letztlich nichts zum Messproblem, das gibt es dort genauso wie in der gewöhnlichen Ein-Teilchen-QM.

  177. #177 Jürgen U
    Münster
    5. Januar 2019

    Noch einmal: es geht nicht um die Mathematik, sondern um Philosophie – z.B. die Rolle der Mathematik im Erkenntnisprozess. Physikalische Grundlage der Bohmschen Mechanik ist die zeitabhängige Schrödingergleichung sowie die Bewegungsgleichung als „Führungsgleichung“ für die Teilchenorte. Philosophische Grundlage ist der Determinismus, der Kausalnexus und der Grundsatz, dass die Dinge unabhängig vom menschlichen Bewusstsein oder irgendeiner Messvorrichtung existieren. Ebenso die synthetische QT von U. Hoyer. Beide kommen zu genau den gleichen Formeln & Vorhersagen wie die konvent. QT – nur sauber abgeleitet und nicht zusammengeschustert. Beschäftigt euch vielleicht mal mit der Geschichte der QT. Schon Planck gab zu, dass er eigentlich gar nicht so richtig verstand, was er da zusammengebaut hatte – er war eben auch Philosoph….

  178. #178 MartinB
    5. Januar 2019

    @JürgenU
    “Noch einmal: es geht nicht um die Mathematik, sondern um Philosophie ”

    Guck mal: Du bist hier mit solchen Klopfern eingestiegen:
    “definierter fester Ort bedeutet, dass das Objekt zu diesem Zeitpunkt keine Geschwindigkeit/Impuls haben kann”
    die wirklich haarsträubender Unsinn sind.
    Na nützt jetzt ein bisschen nachträgliches name-dropping auch nichts mehr. Und wer von Bohm redet, sollte zumindest Probleme wie die Erweiterung auf Feldtheorien oder die preferred-foliation-Problematik erwähnen und nicht einfach so tun, als sei Bohm vollkommen problemlos.
    ” der Grundsatz, dass die Dinge unabhängig vom menschlichen Bewusstsein oder irgendeiner Messvorrichtung existieren.”
    Du kennst aber schon das Kochen-Specker-Theorem?
    https://de.wikipedia.org/wiki/Kochen-Specker-Theorem

  179. #179 Toni
    5. Januar 2019

    @MartinB:

    “Die QFT sagt letztlich nichts zum Messproblem, das gibt es dort genauso wie in der gewöhnlichen Ein-Teilchen-QM.”

    Da gehen offenbar die Meinungen auseinander.

    https://www.quantum-field-theory.net/how-quantum-field-theory-solve-the-measurement-problem/

    https://arxiv.org/abs/1311.0205

  180. #180 MartinB
    5. Januar 2019

    @Toni
    Verstehe ich nicht. Wo steht denn da irgendwas dazu, wann/warum ein zustand kollabiert?

    ” This is called “quantum collapse” and it is not described by the field equations. In fact there is no theory that describes it.”
    sagt nicht gerade, dass das Problem gelöst ist, oder?

  181. #181 Toni
    5. Januar 2019

    @MartinB:

    In seinem Buch schreibt er:

    Resolution. QFT supplies a simple answer for Schrödinger’s cat and
    Einstein’s bomb. The answer, again, is quantum collapse. Quantum collapse
    happens with or without an observer. In the cat scenario, when a quantum is
    emitted from the radioactive sample, it first interacts with all other quanta that
    it encounters, as described by the field equations. These interactions are
    deterministic and reversible. This phase ends when the quantum collapses and
    transfers its energy to an atom in the Geiger counter. This triggers a discharge
    that trips the relay that releases the poison gas that kills the cat. Until then the
    cat is alive; after that the cat is dead. There are no superpositions of states.

    Alles im Sinne von Julian Schwinger Interpretation der Quantenfeldtheorie.

    https://fieldsofcolor.blogspot.com/2015/12/how-quantum-field-theory-solves.html

  182. #182 MartinB
    5. Januar 2019

    @Toni
    Ja, aber es wird nirgends erklärt, wann denn nun ein Zustand kollabiert, es wird einfach gesagt, dass der Zustand kollabiert. Das weicht dem problem schlicht aus.

  183. #183 Toni
    5. Januar 2019

    Nach Schwinger und seinem Schüler gibt es in QFT keine Superposition, deswegen muss man sich nicht um “wann denn nun ein Zustand kollabiert” kümmern.

    That implementation of the principle of superposition to the system as a whole leads to a paradoxical of measurement process and the collapse of the wave function.

    Quantum collapse: The field equations do not tell the whole story; in fact, they don’t tell the most important part of the story. They don’t describe transfer of energy, and without energy transfer nothing of significance can happen, including measurement. When a quantum transfers its energy to an atom it must collapse into that atom. It cannot continue to exist after it has given up its energy. Collapse occurs even if a quantum transfers part of its energy.

    Mit Kollaps in QFT verbindet Schwinger ein energy transfer:

    Wave-function collapse in QM is a collapse of probabilities, while in QFT quantum collapse is a physical event.

    Was Schwinger mit seiner QFT NICHT beschreiben kann, ist, wann dieser Energie-Transfer stattfindet.

    Das ist natürlich nichts weiter als ein Versuch, das Problem innerhalb der QFT zu lösen. Das ist höchstens nur ein kleiner Schritt in die richtige Richtung. Ich überbewerte diese Interpretation nicht.

  184. #184 MartinB
    5. Januar 2019

    @Toni
    Oben hast du noch geschrieben:
    “Das Messproblem, Quantenverschränkung, etc… werden zumindest einigermaßen durch Quantenfeldtheorie beseitigt, wenn man von Felder statt von Teilchen ausgeht”

    Das Schwinger-Zitat finde ich ehrlich gesagt einigermaßen unverständlich. Warum soll es in der QFT keine Überlagerung von Zuständen geben – ein kohärenter Zustand des Photonfeldes ist doch z.B. eine Überlagerung von Zuständen mit unterschiedlicher Teilchenzahl. Ich habe ehrlich gesagt keine Ahnung, wie er das meint – und generell ist es ja oft schwierig/irreführend, Zitate von den Begründern einer Theorie von vor langer Zeit heute zu interpretieren, haben wir neulich auch bei einem Einstein-Zitat gesehen, weil viele Begriffe heute anders benutzt werden.

  185. #185 Toni
    5. Januar 2019

    @MartinB:

    Ich denke, er meint, dass es in seiner Formulierung der QFT der Zustand tot und lebendig bei Schrödinger Katze nicht gibt, sondern wie er sagt, es gibt nur ein energy transfer, dessen Zeitpunkt er nicht bestimmten kann. Aber seine source theory scheint offenbar umstritten zu sein.

  186. #186 Niels
    5. Januar 2019

    @MartinB @Toni

    Das sind doch gar keine Schwinger-Zitate.
    Sondern Zitate von diesem Rodney Brooks, der behauptet, genau zu wissen, wie Schwinger die QFT interpretierte.

    Das macht er daran fest, dass er vor mehr als 50 Jahren mal als undergraduate eine QFT-Vorlesung von Schwinger besucht hat.
    Und schließlich gelte
    In my opinion, if you didn’t learn QFT from Schwinger, you didn’t learn QFT.
    .

    Brooks schrieb dann vor Kurzem, nach seiner Pensionierung als Medizinphysiker(!), ein populärwissenschaftliches Buch über die QFT.

    After a career at the National Institutes of Health (where I published 120 scientific papers and designed the highest resolution PET scanner of its time), I came to realize that this wonderful theory had been forgotten – both by the physics community and the general public
    […]
    I believe this is an intellectual tragedy and is not good for physics or for society. And so I decided, as my retirement mission, to write a book explaining QFT to a lay audience (and also to many physicists)[…]

    .

    Ich kenne sein Buch nicht, aber das hier bisher von ihm Zitierte reißt mich nicht vom Hocker.

    Vor 50 Jahren die Vorlesung eines berühmten Physikers gehört zu haben qualifiziert nicht wirklich automatisch zur Koryphäe.
    (Die Einschätzung, die QFT sei von der physikalischen Gemeinschaft vergessen worden sei, überzeugt mich jetzt auch nicht wirklich von seiner Kompetenz auf diesem Gebiet.
    Ist das nicht eine extrem absurde Idee?)

    Daher glaube ich, dass es ziemlich sinnlos ist, über die Interpretationen dieser (an ein Laienpublikum gerichteten) Rodney-Brooks-Blog-Zitate zu diskutieren.

    Ich habe zwar sehr wenig Ahnung von der QFT [obwohl ich vor weniger als 15 Jahren QFT 1 und(!) 2 gehört habe 😉 (allerdings zugegeben nicht bei Schwinger)].
    Aber ich bin mir doch sehr sicher, dass diese Zitate weder den aktuellen Wissensstand noch den von vor 50 Jahren wiederspiegeln.

  187. #187 Toni
    5. Januar 2019

    @Niels:

    Ich habe sein Buch gelesen und es besteht im wesentlichen aus vielen Zitaten von diversen Physiker. Dass Schwinger QFT anders gesehen hat, ist nicht wirklich die Meinung von seinem Schüler, das ist, wo weit ich beurteilen kann, belegt. Man muss halt das Buch lesen. In seinem Buch wird Feynman heftig angegangen, weil er nicht an Felder geglaubt habe. Das wurde auch in einem Interview von Freeman Dyson (zu sehen auf YouTube) bestätigt.

    Die Source Theory von Schwinger ist ein Versuch die Probleme der QM zu beseitigen. Wie ich schon geschrieben habe, sie ist umstritten und fand nicht wirklich Akzeptanz. Wie gesagt, ich überbewerte Schwinger’s Interpretation nicht. Er reduziert das Mess-Problem auf ein energy transfer, aber er löst es nicht vollständig.

  188. #188 Jürgen U
    Münster
    5. Januar 2019

    Noch ein letzter Satz zu Martin:
    Es ist schade, dass du von all dem, was ich geschrieben habe 2, 3 physikalische Punkte, die eher marginal sind, herausgepickt hast und darauf herumgeritten bist, um meine angebliche Inkompetenz herauszustellen. Der Aggressor bist in diesem Falle du, weil du dich offensichtlich persönlich gekränkt fühlst, wenn man den großen Heisenberg kritisiert (und ich stehe bei Gott nicht alleine damit!). Deine Antworten waren aggressiv & abwertend und du bist mit keinem Wort auf das eingegangen, worauf es mir ankam: Welche Rolle spielen mathematische Aussagen im Erkenntnisprozess? Wie ist die QT begründet und welche philosophischen Implikationen hat sie? Du hast – ich muss es leider sagen – von Philosophie nicht die geringste Ahnung. Das ist die Sache mit dem Tellerrand! Ich gehöre wohl einer aussterbenden Generation an, die nicht autoritätshörig alles frisst, was man ihr vorsetzt. Okay, das soll’s gewesen sein :/:

  189. #189 Karl-Heinz
    5. Januar 2019

    @Jürgen U

    Na ja … du bist aber auch nicht einfach. Du musst schon zugeben, dass du auf bestimmten Gebieten inkompetent bist, oder? Und dass was du über Heisenberg geschrieben hast und wie du es geschrieben hast, ob das was mit Philosophie zu tun hat? Da bin ich mir auch nicht ganz sicher. 😉

  190. #190 MartinB
    6. Januar 2019

    @Niels
    Danke, da war ich wohl mal wieder zu naiv.

    @JürgenU
    Du steigst hier ein mit einem persönlichen Angriff auf Heisenberg. (Zu dem ich schlicht keine Meinung habe – außer ein paar Büchern habe ich nichts von ihm gelesen.) Du untermauerst Deine Meinung mit absolutem Quatsch. Und dann erwartest du, dass ich über diese physikalischen Punkte hinwegsehe (das mit der Unschärfe war aber doch der Kern deines “Arguments”), weil es ja eigentlich um Philosophie geht.

    “Welche Rolle spielen mathematische Aussagen im Erkenntnisprozess? Wie ist die QT begründet und welche philosophischen Implikationen hat sie? ”
    Kann man diskutieren – dazu wäre mein Blogpost über QM und Realität vermutlich der bessere Platz.

    “Du hast – ich muss es leider sagen – von Philosophie nicht die geringste Ahnung”
    Das kannst du sicher gut beurteilen, weil…

    ” Ich gehöre wohl einer aussterbenden Generation an, die nicht autoritätshörig alles frisst, was man ihr vorsetzt. ”
    BINGO! Das Klischee hat ja in deinen Kommentaren noch gefehlt.

  191. #191 MartinB
    6. Januar 2019

    PS
    Das mit dem Mond ist auch deutlich weniger einfach, als deine naive Weltsicht es denkt:
    https://cp3.irmp.ucl.ac.be/~maltoni/PHY1222/mermin_moon.pdf
    Und wie gesagt, ein Blick auf das Kochen-Specker-Theorem schadet auch nicht….

  192. #192 Jürgen U
    6. Januar 2019

    – lol –

  193. #193 Karl-Heinz
    6. Januar 2019

    @Jürgen U

    c(:

  194. #194 Robert
    Neubrandenburg
    12. Juli 2019

    Richtig guter Artikel hat mir sehr geholfen, ich würde sagen Person Robert 1 |verstanden> + 0 |nichtverstanden> = 1

  195. #195 MartinB
    13. Juli 2019

    @Robert
    Freut mich.
    Aber wenn du zwei Quantenzustände addierst, kommt nie einfach ne Zahl raus, für die Wahrscheinlichkeit musst du noch quadrieren…

  196. #196 Emil Dervisevic
    Ulm
    25. April 2020

    Der Artikel ist super.

  197. #197 MartinB
    26. April 2020

    @Emil
    Danke